Download as pdf or txt
Download as pdf or txt
You are on page 1of 263

TABLE OF CONTENTS

Neurology 1.
2.
Embryology
Cellular Function
3. Ascending Spinal Cord Tracts
4. Descending Spinal Cord Tracts
5. Brainstem Anatomy
6. Cerebral Cortex
7. Radiculopathy
8. Spinal Cord Syndromes
9. Invasive Spinal Cord Disease
10. Demyelinating Disease
11. Conduction Physiology
12. Neuromuscular Junction (NMJ)
13. Cranial Nerves 1-6
14. Cranial Nerves 7-12
15. Vision
TABLE OF CONTENTS

Neurology 16.
17.
Auditory Sensation
Vertigo
18. Pediatric Brain Tumors
19. Adult Primary Brain Tumors
20. Ischemic Cerebrovascular Accidents
21. Aneurysms and Intracranial Hemorrhage
22. Cerebellum and Ventricles
23. Diencephalon
24. Basal Ganglia
25. Neurotransmitter Activity in Psychiatric Disease
26. Dementia
27. Headache
28. Seizures
29. Traumatic Brain Injuries
OUTLINE

Neurology:
1. Embryology
● A. Notochord
● B. Neural Tube

Embryology ●


C. Meninges
D. Origins of Nervous Tissue
E. Neural Tube Defects
Neurology: Embryology Bootcamp.com

• Notochord
• Derived from mesoderm
• Stimulation of ectoderm à neuroectoderm
• ~2.5 weeks gestation
• Neural Tube
• Derived from medial ectoderm
• Fuses dorsally à neural tube
• ~3 weeks gestation

https://commons.wikimedia.org/wiki/File:2912_Neurulation-02.jpg
Neurology: Embryology Bootcamp.com

• Epidural Space
• Epidural anesthesia
• Epidural hematoma
• Dura Mater
• Derived from mesoderm
• Subdural Space
• Subdural hematoma
• Arachnoid Mater
• Derived from neural crest cells
• Subarachnoid Space
• Contains CSF
• Lumbar puncture
• Subarachnoid Hemorrhage
• Pia Mater
• Derived from neural crest cells
Neurology: Embryology Bootcamp.com

Origins of Nervous Tissue


Embryologic Vesicle High Yield Derivatives
Origin

Hypothalamus
Diencephalon Thalamus
Pineal gland

Forebrain Prosencephalon

Cerebral hemispheres
Telencephalon
Basal ganglia (not all of it)

Midbrain Mesencephalon Mesencephalon Midbrain

Cerebellum
Metencephalon
Pons
Hindbrain Rhombencephalon
Myelencephalon Medulla
Neurology: Pathophysiology: Brain Bootcamp.com

Embryologic
• Anencephaly Cranial Defects
• Failure of rostral neuropore closure Syndrome Detail Signs and Symptoms
• Elevated AFP
Anencephaly Absence of forebrain Fatal, incompatible with
life

Lissencephaly Lack of cortical sulci and


gyri

Microcephaly
Ventriculomegaly
Hydrocephalus
Craniofacial
abnormalities
Neurology: Pathophysiology: Spinal Cord Bootcamp.com

Spinal Defects
Syndrome Detail Signs and Symptoms

Spina Bifida Occulta Vertebral Defect Patch of hair


Skin dimple

Meningocele Meninges through vertebral


defect

+/- Motor impairment,


developmental delay,
bladder and bowel
incontinence

Myelomeningocele Spinal cord through vertebral


defect https://commons.wikimedia.org/wiki/File:Typesofspinabifida.jpg
≣ ⟽ ⟾
Item 1 of 1 Test Your Knowledge
◽" Mark Difficulty: ✪ Bootcamp.com
Question ID: 0001 Previous Next

A 27-year-old female with a past medical history of generalized anxiety


⚪ A. Failure of closure of the rostral neuropore disorder and recurring seizures that began during adolescence presents
⚪ B. Failure of closure of the caudal neuropore to her obstetrician reporting that she had multiple positive pregnancy tests
⚪ C. Degeneration of anterior horn cells over the course of the past 18 weeks. She states that she has avoided
⚪ D. Protrusion of meninges through a bony defect prenatal care up to this point because she has felt more anxious since
⚪ E. Protrusion of neural tissue through a bony defect she found out she was pregnant. She also states that she has been taking
more of her medications to calm her anxiety. When questioned she
mentions she is particularly concerned of her higher stress levels
precipitating a seizure episode. Her last seizure episode was
approximately 4 years prior to this visit. Her vitals are stable. Prenatal and
fetal assessments are performed and some of the results are shown
below. Which of the following pathologic processes are most consistent
with the patient’s lab and imaging results?

Urine dipstick negative for protein.


Random glucose 108 mg/dL
Fetal ultrasound findings are displayed.
Amniotic fluid index 44cm (reference range <25cm)
Serum alpha fetal protein elevated.

https://commons.wikimedia.org/wiki/File:Ultrasound_Scan_ND_145330_1455290_cr.png
≣ ⟽ ⟾
Item 1 of 1 Test Your Knowledge
◽" Mark Difficulty: ✪ Bootcamp.com
Question ID: 0001 Previous Next

A 27-year-old female with a past medical history of generalized anxiety


" A. Failure of closure of the rostral neuropore disorder and recurring seizures that began during adolescence presents
⚪ B. Failure of closure of the caudal neuropore to her obstetrician reporting that she had multiple positive pregnancy tests
⚪ C. Degeneration of anterior horn cells over the course of the past 18 weeks. She states that she has avoided
⚪ D. Protrusion of meninges through a bony defect prenatal care up to this point because she has felt more anxious since
⚪ E. Protrusion of neural tissue through a bony defect she found out she was pregnant. She also states that she has been taking
more of her medications to calm her anxiety. When questioned she
mentions she is particularly concerned of her higher stress levels
precipitating a seizure episode. Her last seizure episode was
approximately 4 years prior to this visit. Her vitals are stable. Prenatal and
fetal assessments are performed and some of the results are shown
below. Which of the following pathologic processes are most consistent
with the patient’s lab and imaging results?

Urine dipstick negative for protein.


Random glucose 108 mg/dL
Fetal ultrasound findings are displayed
Amniotic fluid index 44cm (reference range <25cm)
Serum alpha fetal protein elevated

https://commons.wikimedia.org/wiki/File:Ultrasound_Scan_ND_145330_1455290_cr.png
OUTLINE

Neurology:
1. Neuronal Cells
● A. Cell Types
● B. Neuronal Anatomy

Cellular ●

C. Neuronal Injury
D. Wallerian Degeneration

Function 2.

Sensory Receptors
A. Pain Fibers
● B. Corpuscles and Disc Receptors
Neurology: Cellular Function Bootcamp.com

• Schwann Cells (PNS)


• 1 cell myelinates 1 axon
• Neural crest cell origin, S100 positive
• Efficient in Wallerian degeneration and regeneration
• Classic pathology associations: GBS, acoustic neuroma
• Oligodendrocytes (CNS)
• 1 cell myelinates many axons
• Neuroectoderm origin
• Not effective in Wallerian degeneration and regeneration (vs Schwann)
• Classic pathology associations: MS, PML, Oligodendroglioma
• Astrocytes
• Assist in maintenance of blood-brain barrier
• Neuroectoderm origin
• Glial Fibrillary Acidic Protein (GFAP) Marker
• Classic pathology association: Glioblastoma multiforme
• Microglia
• Phagocytes of CNS https://commons.wikimedia.org/wiki/File:Oligodendrocyte_HE_stain_
high_mag.jpg
• Mesoderm origin https://commons.wikimedia.org/wiki/File:Anaplastic_astrocytoma_-
• HIV associated dementia _gfap_-_very_high_mag.jpg
Neurology: Cellular Function Bootcamp.com

• Neuron Anatomy
• Soma: Primarily contains cell organelles including Nissl substance
• Axon: Slender, cylindrical projection for conduction action potentials
• Dendrite: Small projections used to receive input, contains Nissl substance
• Nissl Substance = Rough endoplasmic reticulum
• Neuron Motor Physiology
• Kinesin: Motor protein for anterograde transport
• Dynein: Motor protein for retrograde transport
• Neuron Injury
• Cellular edema, peripheral nucleus
• Chromatolysis
https://commons.wikimedia.org/wiki/File:NisslHippo2.jpg

https://commons.wikimedia.org/wiki/File:Blausen_0657_MultipolarNeuron.png
Neurology: Cellular Function Bootcamp.com

• Wallerian Degeneration
• Proximal axonal membrane retained (axonal retraction)
• Distal degeneration and clearing of axonal debris
• Axotomy PNS
• Schwann cells à separate myelin into fragments and promote regeneration
• Macrophages remove axonal debris
• Axotomy CNS
• Oligodendrocytes are inefficient at removing myelin
• Macrophages must cross blood-brain barrier

https://commons.wikimedia.org/wiki/File:Blausen_0657_MultipolarNeuron.png
Neurology: Cellular Function Bootcamp.com

Sensory Receptors of the Nervous System


Sensory Receptor Size, Myelination Primary Sensory High Yield Association
Transmission
A! fibers Myelinated Temperature Fast conduction velocity
Pain (Mechanical) Higher space constant
Lower time constant
C fibers Unmyelinated Pain (General) Slow conduction velocity
Lower space constant
Higher time constant
Neurology: Cellular Function Bootcamp.com

Sensory Receptors of the Nervous System


Sensory Receptor Size, Myelination Primary Sensory High Yield Association
Transmission
Meissner Corpuscles Can be mixed Fine and light touch Located in areas without hair
Low frequency vibration Histology: dermal papillae
Pacinian Corpuscles Large, Myelinated Deep touch and pressure Histology: “Onion-skin”
High frequency vibration
Merkel Discs Large, Myelinated Pressure Located on fingertips and
Proprioception areas hair follicles
Ruffini Corpuscles Large, Myelinated Low frequency vibration Joints
Sustained pressure

https://commons.wikimedia.org/wiki/File:Meissners_Corpuscle_(3384752409).jpg
https://upload.wikimedia.org/wikipedia/commons/5/58/Pacinian_Corpuscles_%281072806656%2
9.jpg
≣ ⟽ ⟾
Item 1 of 1 Test Your Knowledge
◽" Mark Difficulty: ✪✪ Bootcamp.com
Question ID: 0002 Previous Next

A 19 year old male college student reports to the emergency room after
⚪ A. Glial fibrillary acid protein positive stating that he can no longer put up with the “sirens going off in his head”.
⚪ B. Multinucleated giant cells He reports a long history of vertigo since he was a child. He states that he
⚪ C. S100 positivity has tried numerous types of medicine and has seen multiple specialists in
⚪ D. Embryonic glomerular structures the past with no resolve. He also reports having to sit in the front row
⚪ E. Cells with a “fried-egg” appearance on histology recently during lectures on campus because he cannot hear the
professors as well as his classmates. Which of the following pathologic
findings would be most consistent with a biopsy of the tumor(s) causing
this patient’s symptoms?

https://radiopaedia.org/cases/bilateral-acoustic-schwannomas-on-a-background-of-
neurofibromatosis-type-2
≣ ⟽ ⟾
Item 1 of 1 Test Your Knowledge
◽" Mark Difficulty: ✪✪ Bootcamp.com
Question ID: 0002 Previous Next

A 19 year old male college student reports to the emergency room after
⚪ A. Glial fibrillary acid protein positive stating that he can no longer put up with the “sirens going off in his head”.
⚪ B. Multinucleated giant cells He reports a long history of vertigo since he was a child. He states that he
" C. S100 positivity has tried numerous types of medicine and has seen multiple specialists in
⚪ D. Embryonic glomerular structures the past with no resolve. He also reports having to sit in the front row
⚪ E. Cells with a “fried-egg” appearance on histology recently during lectures on campus because he cannot hear the
professors as well as his classmates. Which of the following pathologic
findings would be most consistent with a biopsy of the tumor(s) causing
this patient’s symptoms?

https://radiopaedia.org/cases/bilateral-acoustic-schwannomas-on-a-background-of-
neurofibromatosis-type-2
OUTLINE

1. Ascending Spinal Cord Tracts


Neurology: ●

A. Spinothalamic Tract
B. Dorsal Column-Medial Leminiscus

Ascending ●

2.
C. Spinocerebellar Tract

Anatomical Considerations
Spinal Cord ●

A. Fasciculi of Dorsal Columns
B. Visual Representation of Tracts

Tracts
Neurology: Ascending Spinal Cord Tracts Bootcamp.com

Ascending Tracts
Tract Sensation and Function High Yield Neuroanatomy
Lateral Spinothalamic Pain
Temperature
Travels 1-2 spinal levels superiorly prior to decussation
Pain (General)
Anterior Spinothalamic Crude Touch
Pressure
Neurology: Ascending Spinal Cord Tracts Bootcamp.com

Ascending Tracts
Tract Sensation and Function High Yield Neuroanatomy
Lateral Spinothalamic Pain
Temperature
Travels 1-2 spinal levels superiorly prior to decussation
Pain (General)
Anterior Spinothalamic Crude Touch
Pressure
Dorsal Column-Medial Fine Touch, Low frequency vibration à (Meissner Decussation at medulla
Lemniscus (DCML) corpuscles) Fasciculus gracilis: Approximately T6 and below (located
Proprioception (Muscle spindles) medially in dorsal column)
Pressure and High Frequency Vibrationà (Pacinian) Fasciculus cuneatus: Approximately T5 and above
(located laterally in dorsal column)
Neurology: Ascending Spinal Cord Tracts Bootcamp.com

Ascending Tracts
Tract Sensation and Function High Yield Neuroanatomy
Lateral Spinothalamic Pain
Temperature
Travels 1-2 spinal levels superiorly prior to decussation
Pain (General)
Anterior Spinothalamic Crude Touch
Pressure
Dorsal Column-Medial Fine Touch, Low frequency vibration à (Meissner Decussation at medulla
Lemniscus (DCML) corpuscles) Fasciculus gracilis: Approximately T6 and below (located
Proprioception (Muscle spindles) medially in dorsal column)
Pressure and High Frequency Vibrationà (Pacinian) Fasciculus cuneatus: Approximately T5 and above
(located laterally in dorsal column)
Spinocerebellar Unconscious proprioception All fibers carry data to the ipsilateral cerebellum.
This may involve ”double decussation”.
Neurology: Ascending Spinal Cord Tracts Bootcamp.com

https://commons.wikimedia.org/wiki/File:Spinal_cord_tracts_-_English.svg
≣ ⟽ ⟾
Item 1 of 1 Test Your Knowledge
◽" Mark Difficulty: ✪✪ Bootcamp.com
Question ID: 0003 Previous Next

A 24-year-old female with recently diagnosed with Hashimoto’s thyroiditis


⚪ A. Symmetrical demyelination of fasciculus gracilis presents to the emergency room stating that she has been experiencing
⚪ B. Fluid filled obstructive syrinx of central spinal canal progressively worsening difficulty walking for the past couple of months.
⚪ C. Molecular mimicry to gangliosides of dorsal columns She reports that her feet go numb and she feels like she is losing her
⚪ D. Oligodendrocyte demyelination optic nerve balance. She is very concerned of falling and reports that today she had a
⚪ E. Mitochondrial dysfunction of cells in the prefrontal cortex particularly bad episode of acute numbness of both feet prior to arrival.
This has been difficult for her to cope with as she had previously been a
half marathon runner. She states that she also has had a significant drop
in her productivity and energy. Her vital signs and significant lab studies
are shown below. Which of the following pathologic findings are most
consistent with this presentation?

Temperature: 99.0F (37.2C)


Heart rate: 101 beats per minute
Respiratory rate: 16 breaths per minute
Blood pressure: 98/54

Hemoglobin: 9.4 g/dL


Mean corpuscular volume: 110
Total bilirubin: 1.2 mg/dL (ref. range 0.3-1.0 mg/dL)
Thyroid stimulating hormone: normal
Anti-intrinsic factor antibody: positive
≣ ⟽ ⟾
Item 1 of 1 Test Your Knowledge
◽" Mark Difficulty: ✪✪ Bootcamp.com
Question ID: 0003 Previous Next

A 24-year-old female with recently diagnosed with Hashimoto’s thyroiditis


$ A. Symmetrical demyelination of fasciculus gracilis presents to the emergency room stating that she has been experiencing
⚪ B. Fluid filled obstructive syrinx of central spinal canal progressively worsening difficulty walking for the past couple of months.
⚪ C. Molecular mimicry to gangliosides of dorsal columns She reports that her feet go numb and she feels like she is losing her
⚪ D. Oligodendrocyte demyelination optic nerve balance. She is very concerned of falling and reports that today she had a
⚪ E. Mitochondrial dysfunction of cells in the prefrontal cortex particularly bad episode of acute numbness of both feet prior to arrival.
This has been difficult for her to cope with as she had previously been a
half marathon runner. She states that she also has had a significant drop
in her productivity and energy. Her vital signs and significant lab studies
are shown below. Which of the following pathologic findings are most
consistent with this presentation?

Temperature: 99.0F (37.2C)


Heart rate: 101 beats per minute
Respiratory rate: 16 breaths per minute
Blood pressure: 98/54

Hemoglobin: 9.4 g/dL


Mean corpuscular volume: 110
Total bilirubin: 1.2 mg/dL (ref. range 0.3-1.0 mg/dL)
Thyroid stimulating hormone: normal
Anti-intrinsic factor antibody: positive
OUTLINE

1. Motor Neurons
Neurology: ●

A. Types of Motor Neurons
B. Fundamental Spinal Cord Anatomy

Descending ●

2.
C. Motor Neuron Lesion Presentation

Descending Spinal Cord Tracts


Spinal Cord ●

A. Corticospinal Tract
B. Corticobulbar Tract
● C. Extrapyramidal Tracts
Tracts ● D. Visual Representation of Tracts
Neurology: Descending Spinal Cord Tracts Bootcamp.com

• Gray Matter
• Ventral Horn à !-motor neurons à motor (efferent nerve root fibers)
• Dorsal Horn à sensory neurons à sensory and reflexes (afferent nerve root fibers)
• Lateral Horn à Sympathetics (T1-L2)

Motor Neurons
Type Innervation Significance
!-motor Extrafusal muscle fibers Contraction of skeletal
neurons muscle
"-motor Intrafusal muscle fibers Regulates muscle spindle
neurons sensitivity

https://commons.wikimedia.org/wiki/File:Spinal_nerve.svg
Neurology: Descending Spinal Cord Tracts Bootcamp.com

Motor Neuron Lesion Signs


Upper Lower

Corticospinal tract UMN Anterior horn à NMJ


Hallmark Location Corticobulbar tract UMN LMNs

Hypertonia Hypotonia
Clinical Signs Hyperreflexia Hyporeflexia
Muscle weakness Flaccid paresis
Spastic paresis Muscle atrophy
Babinski sign (upgoing) Fasciculations
Neurology: Descending Spinal Cord Tracts Bootcamp.com

Descending Tracts
Tract Primary Sensation and Function High Yield Neuroanatomy

Lateral Corticospinal Voluntary Muscle Movement (general) Pyramidal decussation at inferior medulla
Terminate in ventral horn
Lower extremity à Lateral
Pyramidal

Anterior Corticospinal Voluntary Muscle Movement (general) Remains ipsilateral in medulla


Decussation at termination in ventral horn at
specific cervical and thoracic segments
Extrapyramidal
Neurology: Descending Spinal Cord Tracts Bootcamp.com

Descending Tracts
Tract Primary Sensation and Function High Yield Neuroanatomy

Lateral Corticospinal Voluntary Muscle Movement (general) Pyramidal decussation at inferior medulla
Terminate in ventral horn
Lower extremity à Lateral
Pyramidal

Anterior Corticospinal Voluntary Muscle Movement (general) Remains ipsilateral


Decussation at termination in ventral horn at specific
cervical and thoracic segments

Corticobulbar Voluntary Muscle Movement (head and neck) Contralateral innervation only with:
-Inferior quadrant of face CN VII
-CN XII
Extrapyramidal
Neurology: Descending Spinal Cord Tracts Bootcamp.com

Descending Tracts
Tract Primary Sensation and Function High Yield Neuroanatomy

Lateral Corticospinal Voluntary Muscle Movement (general) Pyramidal decussation at inferior medulla
Terminate in ventral horn
Lower extremity à Lateral
Pyramidal

Anterior Corticospinal Voluntary Muscle Movement (general) Remains ipsilateral


Decussation at termination in ventral horn at
specific cervical and thoracic segments

Corticobulbar Voluntary Muscle Movement (head and neck) Contralateral innervation only with
-Inferior quadrant of face CN VII
-CN XII

Reticulospinal Vestibulospinalà balance and posture Rubrospinal à Red nucleus


Extrapyramidal

Rubrospinal Tectospinal à Head movements per vision


Tectospinal
Vestibulospinal
Neurology: Spinal Cord Tracts Bootcamp.com

https://commons.wikimedia.org/wiki/File:Spinal_cord_tracts_-_English.svg
Neurology: Spinal Cord Tracts Bootcamp.com

A
PS

PS
A

ASA

Image Credit: Wikipedia


≣ ⟽ ⟾
Item 1 of 1 Test Your Knowledge
◽" Mark Difficulty: ✪✪✪ Bootcamp.com
Question ID: 0004 Previous Next

A 17-year-old male with a history of Marfan syndrome is brought to the


⚪ A. Inferiorly directed left lens dislocation emergency department by his parents for acute onset neck swelling and
⚪ B. Left sided deviation of the tongue headache that started 45 minutes prior to arrival. His father reports that
⚪ C. Fasciculations of the left gastrocnemius muscle his son was riding his bike at which time he hit a curb and fell, bracing his
⚪ D. Atrophy of the tongue fall with his right arm. A cervical spine collar is placed for immobilization.
⚪ E. Bilateral abducens nerve palsy He also states that his son has a history of seasonal allergies and has
never had swelling similar to this in the past. On physical exam he has a
heart rate of 103 beats per minute and his vitals signs are otherwise
stable. The chest shows a pectus deformity and is otherwise
unremarkable. The right upper extremity and left knee have minor
lacerations. Muscle strength is 5/5 in the upper and lower extremities.
Funduscopic exam is unremarkable with no evidence of optic disc edema.
There is also left eye drooping noted and an apparent left sided 3.5 cm
neck mass. During the exam, the patient states that he is hearing an
intermittent ”buzzing sound” in the room that is not apparently heard by
staff. Imaging studies later reveal a left sided internal carotid artery
dissection. Which of the following additional findings would be most
consistent with this patient’s presentation?
≣ ⟽ ⟾
Item 1 of 1 Test Your Knowledge
◽" Mark Difficulty: ✪✪✪ Bootcamp.com
Question ID: 0004 Previous Next

A 17-year-old male with a history of Marfan syndrome is brought to the


⚪ A. Inferiorly directed left lens dislocation emergency department by his parents for acute onset neck swelling and
$ B. Left sided deviation of the tongue headache that started 45 minutes prior to arrival. His father reports that
⚪ C. Fasciculations of the left gastrocnemius muscle his son was riding his bike at which time he hit a curb and fell, bracing his
⚪ D. Significant atrophy of the tongue musculature fall with his right arm. A cervical spine collar is placed for immobilization.
⚪ E. Bilateral abducens nerve palsy He also states that his son has a history of seasonal allergies and has
never had swelling similar to this in the past. On physical exam he has a
heart rate of 103 beats per minute and his vitals signs are otherwise
stable. The chest shows a pectus deformity and is otherwise
unremarkable. The right upper extremity and left knee have minor
lacerations. Muscle strength is 5/5 in the upper and lower extremities.
Funduscopic exam is unremarkable with no evidence of optic disc edema.
There is also left eye drooping noted and an apparent left sided 3.5 cm
neck mass. During the exam, the patient states that he is hearing an
intermittent ”buzzing sound” in the room that is not apparently heard by
staff. Imaging studies later reveal a left sided internal carotid artery
dissection. Which of the following additional findings would be most
consistent with this patient’s presentation?

https://commons.wikimedia.org/wiki/File:Gray794.png
OUTLINE

Neurology:
1. Brainstem Anatomy
● A. Midbrain
● B. Pons

Brainstem ●


C. Tectum
D. Medulla
D. Introduction to the Rule of 4’s

Anatomy 2. Rule of 4’s


● A. Introductory Review
Neurology: Brainstem Anatomy Bootcamp.com

Brainstem
Syndrome Embryonic Blood Supply Significant Structures
Origin

CN III-IV nuclei
Substantia Nigra
Branches of Posterior Cerebral Artery Red Nucleus
Midbrain Mesencephalon
and Superior Cerebellar Artery Pretectal Area
Medial Longitudinal Fasciculus
Superior Cerebellar Peduncles
Neurology: Brainstem Anatomy Bootcamp.com

• Tectum of Brainstem
• Superior Colliculi à Lateral Geniculate Body àVision
• Inferior Colliculi à Medial Geniculate Body à Auditory
Neurology: Brainstem Anatomy Bootcamp.com

Brainstem
Syndrome Embryonic Blood Supply Significant Structures
Origin

CN III-IV nuclei
Substantia Nigra
Branches of Posterior Cerebral Artery Red Nucleus
Midbrain Mesencephalon
and Superior Cerebellar Artery Pretectal Area
Medial Longitudinal Fasciculus
Superior Cerebellar Peduncles

Metencephalon Ventral: Basilar Artery CN V-VIII* nuclei


Pons (hindbrain) Lateral: Anterior Inferior Cerebellar Reticular Activating System
Artery Medial Longitudinal Fasciculus
Neurology: Brainstem Anatomy Bootcamp.com

Brainstem
Syndrome Embryonic Blood Supply Significant Structures
Origin

CN III-IV nuclei
Substantia Nigra
Branches of Posterior Cerebral Artery Red Nucleus
Midbrain Mesencephalon
and Superior Cerebellar Artery Pretectal Area
Medial Longitudinal Fasciculus
Superior Cerebellar Peduncles

Metencephalon Ventral: Basilar Artery CN V-VIII* nuclei


Pons (hindbrain) Lateral: Anterior Inferior Cerebellar Reticular Activating System
Artery Medial Longitudinal Fasciculus

CN VIII* nuclei
Medial: Anterior Spinal Artery and/or CN IX-XII nuclei
Myelencephalon Vertebral Arteries Area Postrema
Medulla (hindbrain) Lateral: Posterior Inferior Cerebellar Inferior cerebellar peduncules
Artery
Neurology: Brainstem Anatomy Bootcamp.com

Corticobulbar Tract
• Rule 1:
• CN I and II NOT in the midbrain
Lesions
• CN III-IV nuclei in midbrain
• CN V-VIII* nuclei in pons
• CN VIII*-XII nuclei in medulla Injury Site
• Rule 2:
• CN III, IV, VI, XII in midline (divisible by 12)
• Rule 3:
• 4 primary midline structures begin with “m”
• Motor pathway (CST)
• Medial lemniscus (dorsal column pathway) Bulbar Palsy LMN
• Medial longitudinal fasciculus (does not extend into medulla)
• Motor Nuclei of cranial nerves (CN III, IV, V, VI, VII, IX, X, XI XII)
• Rule 4:
• 4 lateral or side structures begin with “s”
• Spinothalamic pathway
Pseudobulbar
• Sensory Nuclei of CN V
UMN


Sympathetic pathway
Spinocerebellar tract
Palsy
≣ ⟽ ⟾
Item 1 of 1 Test Your Knowledge
◽" Mark Difficulty: ✪✪ Bootcamp.com
Question ID: 0005 Previous Next

A laboratory scientist is attempting to test whether dopaminergic depletion


⚪ A. Mammillary body in mice via an intra-striatal infusion of 6-hydroxydopamine (6-OHDA)
⚪ B. Medial temporal lobe produces similar effects to those seen in the setting of Parkinson Disease
⚪ C. Dorsal medulla in humans. 6-OHDA is known to be neurotoxic to dopaminergic and
⚪ D. Ventral midbrain noradrenergic neural tissue. The researcher uses desipramine to
⚪ E. Substantia gelatinosa selectively protect noradrenergic receptors in select mice. Object
recognition time and cognitive testing using rodent variants of tests used
in humans were evaluated to determine if mice experienced similar effects
to humans with Parkinson Disease. Mice were also given physostigmine
and rivastigmine to evaluate the nature of reversal of disease process.
Which neuroanatomical location is most likely directly impacted by the
neurotoxicity of 6-OHDA?
≣ ⟽ ⟾
Item 1 of 1 Test Your Knowledge
◽" Mark Difficulty: ✪✪ Bootcamp.com
Question ID: 0005 Previous Next

A laboratory scientist is attempting to test whether dopaminergic depletion


⚪ A. Mammillary body in mice via an intra-striatal infusion of 6-hydroxydopamine (6-OHDA)
⚪ B. Medial temporal lobe produces similar effects to those seen in the setting of Parkinson Disease
⚪ C. Dorsal medulla in humans. 6-OHDA is known to be neurotoxic to dopaminergic and
" D. Ventral midbrain noradrenergic neural tissue. The researcher uses desipramine to
⚪ E. Substantia gelatinosa selectively protect noradrenergic receptors in select mice. Object
recognition time and cognitive testing using rodent variants of tests used
in humans were evaluated to determine if mice experienced similar effects
to humans with Parkinson Disease. Mice were also given physostigmine
and rivastigmine to evaluate the nature of reversal of disease process.
Which neuroanatomical location is most likely directly impacted by the
neurotoxicity of 6-OHDA?
OUTLINE

Cerebral
1. Cerebral Cortex
● A. Frontal Lobe
● B. Temporal Lobe

Cortex ●


C. Parietal Lobe
D. Occipital Lobe
E. Internal Capsule
Neurology: Cerebral Cortex Bootcamp.com

• Frontal Lobe
• Precentral Gyrusà Primary Motor Cortex
• Prefrontal Cortex à Personality, concentration, judgement
• Front Eye Fields à Conjugate gaze contralateral
• Broca's Area (dominant) à Speech production
• Lesion to Frontal Lobe
• Precentral Gyrusà Contralateral motor
• Prefrontal Cortex à Disinhibition
• Front Eye Fields à Eye deviation to the side of lesion
• Broca’s Area (dominant) à Expressive aphasia
• Blood Supply: Medial: ACA, Lateral: MCA
https://commons.wikimedia.org/wiki/File:Human_brain_frontal_(coronal)_section.JPG
https://commons.wikimedia.org/wiki/File:FrontalCaptsLateral.png
Neurology: Cerebral Cortex Bootcamp.com

• Temporal Lobe
• Primary Auditory Cortex, input from medial geniculate body
• Wernicke’s Area (dominant) à Understanding speech
• Amygdalaà Memory, decision making, emotions
• Optic Radiations à Meyer’s Loop
• Lesion to Temporal Lobe
• Primary Auditory Cortex à ipsilateral hearing deficits
• Wernicke’s Area (dominant) à Receptive aphasia
• Amygdalaà Kluver-Bucy Syndrome
• Optic Radiations à Contralateral Superior quadrantanopia
• Blood Supply à Mostly MCA https://commons.wikimedia.org/wiki/File:Gray_718-amygdala.png
Neurology: Cerebral Cortex Bootcamp.com

• Parietal Lobe
• Postcentral gyrusà Primary Somatosensory Cortex
• Angular Gyrus à Visual Cortex
• Optic Radiations à Superior Loop
• Lesion to Parietal Lobe
• Primary Somatosensory Cortex à Contralateral sensory loss
• Visual Association Cortex à Gerstmann Syndrome (dominant),
Hemineglect (nondominant)
• Arcuate Fasciculus à Conduction Aphasia
• Optic Radiations à Contralateral Inferior quadrantopsia
• Blood Supply à Medial: ACA, Lateral: MCA https://commons.wikimedia.org/wiki/File:Human_brain_frontal_(coronal)_section.JPG
Neurology: Cerebral Cortex Bootcamp.com

• Occipital Lobe
• Primary Visual Cortex, input from lateral geniculate body
• Lesion to Occipital Lobe
• Primary Visual Cortex à Contralateral homonymous hemianopia, macular
sparing
• Blood Supply à PCA
Neurology: Cerebral Cortex Bootcamp.com

• Internal Capsule
• Anterior Limb:
1. Ascending sensory fibers: Thalamocortical tracts

• Posterior Limb:
1. Descending motor fibers: Corticospinal tract
2. Ascending sensory fibers: Thalamocortical and somatosensory tracts (acoustic, visual)

• Genu:
1. Descending motor fibers: Corticobulbar tract

• Blood Supply à Lenticulostriate arteries


≣ ⟽ ⟾
Item 1 of 1 Test Your Knowledge
◽" Mark Difficulty: ✪✪ Bootcamp.com
Question ID: 0006 Previous Next

A 77 year old right-handed female with a past medical history of


⚪ A. Right frontal lobe hypertension and hyperlipidemia presents to the emergency department
⚪ B. Right parietal lobe from a retired living facility for unusual behavior. Paramedics report that
⚪ C. Left parietal lobe the patient was playing a card game with residents of her facility and was
⚪ D. Left occipital lobe unable to hold her cards in her hands. Staff from her living facility also
⚪ E. Right occipital lobe report that she was unable to sign documentation over the past couple of
weeks saying that she ”could not remember how to write her signature”.
On physical exam cranial nerves III-XII are grossly intact with no deficits
in visual acuity. Reflexes are 2+ bilaterally and muscle strength is 5/5
bilaterally in the upper and lower extremities with the exception of
handgrip strength testing which was deferred by the patient. Sensory
deficits are present to touch and vibration along the right extremities. No
tremors are noted.

A mini mental status exam is preformed of which deficits are noted when
the patient is asked to count backward by sevens. The patient is also
unable to hold a pencil in her right hand. No deficits are appreciated in
speech or prosody.

A CT imaging study of the head reveals a small to medium-sized mass in


the cerebral cortex. Which of the following locations of the mass are most
consistent with the patient’s presentation?
≣ ⟽ ⟾
Item 1 of 1 Test Your Knowledge
◽" Mark Difficulty: ✪✪ Bootcamp.com
Question ID: 0006 Previous Next

A 77-year-old right-handed female with a past medical history of


⚪ A. Right frontal lobe hypertension and hyperlipidemia presents to the emergency department
⚪ B. Right parietal lobe from a retired living facility for unusual behavior. Paramedics report that
$ C. Left parietal lobe the patient was playing a card game with residents of her facility and was
⚪ D. Left occipital lobe unable to hold her cards in her hands. Staff from her living facility also
⚪ E. Right occipital lobe report that she was unable to sign documentation over the past couple of
weeks saying that she ”could not remember how to write her signature”.
On physical exam cranial nerves III-XII are grossly intact with no deficits
in visual acuity. Reflexes are 2+ bilaterally and muscle strength is 5/5
bilaterally in the upper and lower extremities with the exception of
handgrip strength testing which was deferred by the patient. Sensory
deficits are present to touch and vibration along the right extremities. No
tremors are noted.

A mini mental status exam is preformed of which deficits are noted when
the patient is asked to count backward by sevens. The patient is also
unable to hold a pencil in her right hand. No deficits are appreciated in
speech or prosody.

A CT imaging study of the head reveals a small to medium-sized mass in


the cerebral cortex. Which of the following locations of the mass are most
consistent with the patient’s presentation?
OUTLINE

1. General Spinal Cord Function


Neurology: ●

A. Anatomical Considerations
B. Reflex by Level

Radiculopathy 2. Spinal Cord Impingement


● A. Vertebral Disc Herniation
● B. Radiculopathy by Level
● C. Spinal Stenosis
● D. Neuropathic vs Vascular Claudication
Neurology: Anatomy & Histology Bootcamp.com

• Spinal Nerve Roots


• C1-C7 exit ABOVE corresponding vertebra
• C8 and caudal exit BELOW corresponding vertebra

• Spinal Cord Anatomical Considerations


• Terminates at L1-L2 in adults
• Iliac crest at the level of L4 as landmark for lumbar puncture
Neurology: Anatomy & Histology Bootcamp.com

Reflex and Spinal Level


Reflex Primary Spinal Level
Muscle Tested
Biceps Biceps brachii C5-C6

Brachioradalis Brachioradalis C5-C6

Triceps Triceps C7-C8

Cremasteric Cremaster L1-L2

Patellar Quadriceps L2-L4

Achilles Gastrocnemius S1-S2

Bulbospongiosus Bulbospongiosus S2-S4

Sphincteric External Anal S2-S4


Sphincter
Neurology: Anatomy & Histology Bootcamp.com

• Spinal Disc Herniation


• L5-S1 is the most common site
• Affected nerve root usually 1 segment below
• Ex. L5-S1 disc herniation à S1 radiculopathy

https://commons.wikimedia.org/wiki/File:Lagehernia.png
https://commons.wikimedia.org/wiki/File:Blausen_0484_HerniatedLumbarDisc.png
Neurology: Anatomy & Histology Bootcamp.com

Radiculopathy
Level Dermatome Classic Finding

Anterolateral upper arm


C5 Shoulder
Loss of Biceps Reflex

C7 Middle finger Loss of Triceps Reflex

Anteromedial leg
Medial malleolus
L4 Knee Loss of Patellar Reflex
Anterior thigh

Most digits of foot


L5 Anterior leg Foot Drop
Anterolateral thigh

Lateral Malleolus Loss of Achilles Reflex


S1 Posterolateral leg and thigh Inability to “toe walk” https://commons.wikimedia.org/wiki/File:Grant_1962_663.png
Neurology: Anatomy & Histology Bootcamp.com

Claudication
• Spinal Stenosis Neuropathic
• Neuropathic claudication (Pseudo) Vascular
(Pseudo)
• Affected by changes in posture
Localization Bilateral Unilateral
• Back pain (classically low back)
• Worse in extension
Back, radiating down
• Initial test à Radiography spine Pain Location
proximal lower extremities
Distal lower extremity

• Confirmatory test à MRI spine


Type of Pain Sharp, stabbing, radiating Tight, crampy

Worse Spinal extension Movement

Better Spinal flexion Rest

Initial Diagnostic
Radiography of spine Ankle Brachial Index
Study
≣ ⟽ ⟾
Item 1 of 1 Test Your Knowledge
◽" Mark Difficulty: ✪ Bootcamp.com
Question ID: 0007 Previous Next

A 29-year-old male with no significant past medical history reports


⚪ A. Left L4 nerve root shooting, “electric-like” pain down the left buttock and lower extremity
⚪ B. Right L4 nerve root when sitting for the past two weeks. He also states that occasionally when
⚪ C. Left L5 nerve root he attempts to run for more than five minutes, the pains progressively
⚪ D. Left S1 nerve root worsen to the point where he is unable to walk. Pertinent findings on
⚪ E. Left S2 nerve root physical exam include 2+ reflexes at the patellar tendon bilaterally and
right Achilles tendon. No reflex was obtained during testing of the left
Achilles tendon. The patient is able to perform heel-walk, although he is
unable to stand on the toes. There is also a loss of sensation to touch and
vibration at the lateral malleolus. No pain is appreciated when palpating
the lower back and there is no evidence of any “step off” signs.

Lumbar radiography and MRI are shown.

Which of the following nerve roots are most likely involved in the cause of
this patient’s symptoms?

Image Credit: Radiopaedia


Image Credit: wikiradiography
≣ ⟽ ⟾
Item 1 of 1 Test Your Knowledge
◽" Mark Difficulty: ✪ Bootcamp.com
Question ID: 0007 Previous Next

A 29-year-old male with no significant past medical history reports


⚪ A. Left L4 nerve root shooting, “electric-like” pain down the left buttock and lower extremity
⚪ B. Right L4 nerve root when sitting for the past two weeks. He also states that occasionally when
⚪ C. Left L5 nerve root he attempts to run for more than five minutes, the pains progressively
" D. Left S1 nerve root worsen to the point where he is unable to walk. Pertinent findings on
⚪ E. Left S2 nerve root physical exam include 2+ reflexes at the patellar tendon bilaterally and
right Achilles tendon. No reflex was obtained during testing of the left
Achilles tendon. The patient is able to perform heel-walk, although he is
unable to stand on the toes. There is also a loss of sensation to touch and
vibration at the lateral malleolus. No pain is appreciated when palpating
the lower back and there is no evidence of any “step off” signs.

Lumbar radiography and MRI are shown.

Which of the following nerve roots are most likely involved in the cause of
this patient’s symptoms?

Image Credit: Radiopaedia


Image Credit: wikiradiography
OUTLINE

1. Spinal Cord Syndromes


Neurology: ●

A. Anterior Cord Syndrome
B. Posterior Cord Syndrome

Spinal Cord
● C. Central Cord Syndrome
● D. Brown-Sequard Syndrome
● E. Cauda Equina Syndrome

Syndromes ● F. Conus Medullaris Syndrome


Neurology: Spinal Cord Syndromes Bootcamp.com

https://commons.wikimedia.org/wiki/File:Spinal_cord_tracts_-_English.svg
Neurology: Spinal Cord Syndromes Bootcamp.com

https://commons.wikimedia.org/wiki/File:Cord_s.svg

Spinal Cord Syndromes


Syndrome Blood Supply Classic Causes Presentation

Anterior Spinal Artery: Anterior 2/3 Vertebral Burst Fracture Bilateral loss of pain and temperature
Anterior Artery of Adamkiewicz: Primary Blood Supply
Below T8 in Spinal Cord
Hypoperfusion
Aortic Surgery
Vibration and proprioception spared
Muscle weakness
Neurology: Spinal Cord Syndromes Bootcamp.com

https://commons.wikimedia.org/wiki/File:Cord_s.svg

Spinal Cord Syndromes Posterior Cord Syndrome

Syndrome Blood Supply Classic Causes Presentation

Anterior Spinal Artery: Anterior 2/3 Vertebral burst fracture Bilateral loss of pain and temperature
Anterior Artery of Adamkiewicz: Primary Blood
Supply Below T8 in Spinal Cord
Hypoperfusion
Aortic Surgery
Vibration and proprioception spared
Muscle weakness

Posterior Spinal Arteries: Posterior 1/3 Tabes Dorsalis


Loss of vibration and proprioception
Posterior bilaterally SCD
Friedreich Ataxia
below
Neurology: Spinal Cord Syndromes Bootcamp.com

https://commons.wikimedia.org/wiki/File:Cord_s.svg

Spinal Cord Syndromes


Syndrome Important Anatomy Classic Causes Presentation

Anterior Spinal Artery: Anterior 2/3 Vertebral burst fracture Bilateral loss of pain and temperature
Anterior Artery of Adamkiewicz: Primary Blood
Supply Below T8 in Spinal Cord
Hypoperfusion
Aortic Surgery
Vibration and proprioception spared
Muscle weakness

Posterior Spinal Arteries: Posterior 1/3 Tabes Dorsalis


Loss of vibration and proprioception
Posterior bilaterally SCD
Friedreich Ataxia
below

Elderly patient
Spinothalamic and portions of anterior Syringomyelia
Bilateral loss of pain and temperature
Central corticospinal tracts traversing anterior white Whiplash (hyperextension)
Muscle weakness (arm>leg)
commissure
Neurology: Spinal Cord Syndromes Bootcamp.com

https://commons.wikimedia.org/wiki/File:Brown-Sequard.svg
https://commons.wikimedia.org/wiki/File:Cord_s.svg

Spinal Cord Syndromes


Syndrome Important Anatomy Classic Causes Presentation

Anterior Spinal Artery: Anterior 2/3 Vertebral burst fracture Bilateral loss of pain and temperature
Anterior Artery of Adamkiewicz: Primary Blood
Supply Below T8 in Spinal Cord
Hypoperfusion
Aortic Surgery
Vibration and proprioception spared
Muscle weakness

Posterior Spinal Arteries: Posterior 1/3 Tabes Dorsalis


Loss of vibration and proprioception
Posterior bilaterally SCD
Friedreich Ataxia
below

Elderly patient
Spinothalamic and portions of anterior Syringomyelia
Bilateral loss of pain and temperature
Central corticospinal tracts traversing anterior white
commissure
Whiplash (hyperextension)
Muscle weakness (arm>leg)

Brown Spinothalamic tract travels superiorly 1-2 Penetrating Injury (stab


Ipsilateral LMN at level Ipsilateral UMN
below level
levels prior to decussation. wound)
Sequard Contralateral pain/temp loss 1-2 below
Neurology: Spinal Cord Syndromes Bootcamp.com

Distal Spinal Cord Injury


Syndrome Level Affected Distinguishing Features

Conus Medullaris T12-L2 Acute onset sphincter incontinence


Bilateral saddle anesthesia
Tends to be less severe pain
Mix of UMN and LMN features

Cauda Equina Caudal to L2 Gradual onset sphincter incontinence


Unilateral saddle anesthesia
Tends to be more severe pain
Primarily LMN features
≣ ⟽ ⟾
Item 1 of 1 Test Your Knowledge
◽" Mark Difficulty: ✪✪ Bootcamp.com
Question ID: 0008 Previous Next

A 36-year-old male is rushed to the emergency room after being found


unconscious outside for an undetermined length of time. His temperature
is 96F, blood pressure 102/56, heart rate 106 beats per minute, and
respiratory rate is 16 breaths per minute. He is saturating 95% on room
air. The patient is found to have a bullet entry wound in the left flank
proceeding medially with acute angulation in near proximity to the spinal
cord. Which of the following features would be least consistent with this
presentation?

⚪ A. Loss of all sensation to innervated structures ipsilateral at the level of the lesion
⚪ B. Left-sided hyperreflexia and muscle weakness below the level of the lesion
⚪ C. Left-sided loss of proprioception and vibration sense below the level of the lesion
⚪ D. Right-sided loss of temperature and pain sensation below the level of the lesion
⚪ E. Left-sided pupillary contraction and drooping of the eyelid
≣ ⟽ ⟾
Item 1 of 1 Test Your Knowledge
◽" Mark Difficulty: ✪✪ Bootcamp.com
Question ID: 0008 Previous Next

A 36-year-old male is rushed to the emergency room after being found


unconscious outside for an undetermined length of time. His temperature
is 96F, blood pressure 102/56, heart rate 106 beats per minute, and
respiratory rate is 16 breaths per minute. He is saturating 95% on room
air. The patient is found to have a bullet entry wound in the left flank
proceeding medially with acute angulation in near proximity to the spinal
cord. Which of the following features would be least consistent with this
presentation?

⚪ A. Loss of all sensation to innervated structures ipsilateral at the level of the lesion
⚪ B. Left-sided hyperreflexia and muscle weakness below the level of the lesion
⚪ C. Left-sided loss of proprioception and vibration sense below the level of the lesion
⚪ D. Right-sided loss of temperature and pain sensation below the level of the lesion
" E. Left-sided pupillary contraction and drooping of the eyelid

https://commons.wikimedia.org/wiki/File:Brown-Sequard.svg
OUTLINE

1. Infectious
Neurology: ●

A. Tetanospasmin Toxin
B. Rabies Virus

Invasive ●


C. Poliomyelitis
D. Varicella Zoster Virus
E. Neurosyphilis

Spinal Cord 2. Other


● A. Spinal Muscular Atrophy
Disease
Neurology: Invasive Spinal Cord Disease Bootcamp.com

• Mechanism of Action
• Tetanospasmin travels à Renshaw cells in spinal cord
• Cleaves SNARE proteinsà ↓ inhibitory neurotransmitter
• GABA, Glycine are primary inhibitory neurotransmitters https://en.wikipedia.org/wiki/Tetanus#/media/File:Neur
otransmitter_vesicle_before_and_after_exposure_to_
• !-motor neurons unregulated Tetanus_Toxin.jpg

• Clinical Signs and Symptoms


• Spastic paralysis
• Trismus (Lock jaw)
• Risus Sardonicus
• Opisthotonos https://en.wikipedia.org/wiki/Tetanus#/m

• Foul smelling umbilical stump (neonates) edia/File:Opisthotonus_in_a_patient_suff


ering_from_tetanus_-
_Painting_by_Sir_Charles_Bell_-
_1809.jpg
Neurology: Invasive Spinal Cord Disease Bootcamp.com

• Mechanism of Action
• Rabies virus binds to nicotinic acetylcholine receptor
• Retrograde migration (dynein) à Brainà Encephalitis
• Long incubation period
• Reservoir
• Worldwide: Rabid dogs
• U.S.: Bats (also skunks, raccoons, and other animals)
• Clinical Signs and Symptoms
• Nonspecific overall
• Hydrophobia
• Photophobia
• Paralysis
Neurology: Invasive Spinal Cord Disease Bootcamp.com

• Classic Presentations of Paralytic Poliomyelitis


• Unvaccinated child
• Classically affects the proximal muscles
• Severe muscle pain
• Ascending asymmetric flaccid paralysis (LMN)
• Remember your LMN Signs
• Fasciculations
• Hyporeflexia
• Hypotonia
• Muscle atrophy
Image credit:
https://en.wikipedia.org/wiki/Polio#/media/File:Polio_spinal_diagram-en.svg
Neurology: Invasive Spinal Cord Disease Bootcamp.com

• Varicella Zoster Virus


• Primary infection à chicken pox
• Remains dormant in dorsal root ganglia
• Reactivation à shingles
• Reactivation classically in immunocompromised patients
• Classic Presentations of Shingles
• Painful erythematous rash à vesicular
• Dermatomal distribution, does NOT cross midline
• Variants
• Herpes Ophthalmicus
• Herpes Oticus (Ramsay Hunt Syndrome)
• Treatment
• Acyclovir, Valacyclovir
• Post-herpetic neuralgia major complication to untreated https://en.wikipedia.or
https://en.wikipedia.org https://en.wikipedia.org/wiki/Herpes_zoster_ophthalmicus https://en.wikipedia.org/wiki/Shingles#/media/File:Shingles.JPG g/wiki/Shingles#/medi
/wiki/Trigeminal_nerve #/media/File:Trigeminal_herpes_with_uveitis_and_keratitis
a/File:Herpes_zoster_
.jpg
chest.png
Neurology: Invasive Spinal Cord Disease Bootcamp.com

• Classic Presentations of Neurosyphilis


• Immunocompromised patient or poor access to medical care
• Possible new onset dementia at an early age
• Tabes Dorsalis à broad-based ataxia, positive Romberg
• Loss of proprioception
• Areflexia
• Argyll Robertson Pupil
• Light-near dissociation
• Classically associated with neurosyphilis

https://commons.wikimedia.org/wiki/File:Argyll_Robertson_pupil_light_reflex_vs_accommodation_reflex.jpg
Neurology: Invasive Spinal Cord Disease Bootcamp.com

• Classic Presentation of Werdnig-Hoffman Disease (SMA Type I)


• Onset between 0-6 months of age, death by 2 years of age
• Symmetric flaccid paralysis
• Cranial nerves classically spared (normal eye movements)
• Lower motor neuron disease
• General Information
• Autosomal recessive mutation in SMN1 gene, chromosome 5
• Muscle biopsy: Atrophy of motor units

https://commons.wikimedia.org/wiki/File:Polio_spinal_diagram-en.svg
≣ ⟽ ⟾
Item 1 of 1 Test Your Knowledge
◽" Mark Difficulty: ✪✪ Bootcamp.com
Question ID: 0009 Previous Next

A researcher is attempting to understand the pathogenesis of the Rabies


virus. It is thought that the Rabies virus has interactions with p75
neurotrophin receptors (p75NTR) at peripheral neurons. This interaction
has been postulated to allow the virus to enter the neuronal tissue and
travel along the axon. It was found that the Rabies virus transport kinetics
revealed a preference for transport in p75NTR-dependent compartments
when compared to p75NTR-independent compartments. The researcher
attempts to study the transport kinetics of the Rabies virus when adding a
cell permeable molecular target, TAT-Pep50 in mice. Assuming TAT-
Pep50 administration demonstrated an overall decrease in mortality in
mice infected with rabies virus, which of the following would best explain
it’s mechanism of action?
⚪ A. Upregulation of dynein motor proteins along axonal microtubules in the central nervous system
⚪ B. Upregulation of dynein motor proteins along axonal microtubules in the peripheral nervous system
⚪ C. Upregulation of p75NTRs at peripheral nerves
⚪ D. Upregulation of acetylcholine receptors in peripheral nerves
⚪ E. Upregulation of ATPase inhibitors selective to dynein motor proteins at sites of high p75NTR expression
⚪ F. Upregulation of ATPase inhibitors selective to kinesin motor proteins at sites of high p75NTR expression
≣ ⟽ ⟾
Item 1 of 1 Test Your Knowledge
◽" Mark Difficulty: ✪✪ Bootcamp.com
Question ID: 0009 Previous Next

A researcher is attempting to understand the pathogenesis of the Rabies


virus. It is thought that the Rabies virus has interactions with p75
neurotrophin receptors (p75NTR) at peripheral neurons. This interaction
has been postulated to allow the virus to enter the neuronal tissue and
travel along the axon. It was found that the Rabies virus transport kinetics
revealed a preference for transport in p75NTR-dependent compartments
when compared to p75NTR-independent compartments. The researcher
attempts to study the transport kinetics of the Rabies virus when adding a
cell permeable molecular target, TAT-Pep50 in mice. Assuming TAT-
Pep50 administration demonstrated an overall decrease in mortality in
mice infected with rabies virus, which of the following would best explain
it’s mechanism of action?
⚪ A. Upregulation of dynein motor proteins along axonal microtubules in the central nervous system
⚪ B. Upregulation of dynein motor proteins along axonal microtubules in the peripheral nervous system
⚪ C. Upregulation of p75NTRs at peripheral nerves
⚪ D. Upregulation of acetylcholine receptors in peripheral nerves
$ E. Upregulation of ATPase inhibitors selective to dynein motor proteins at sites of high p75NTR expression
⚪ F. Upregulation of ATPase inhibitors selective to kinesin motor proteins at sites of high p75NTR expression
OUTLINE

1. Demyelinating Disease

Neurology: ●

A. Guillain-Barre Syndrome
B. Multiple Sclerosis
● C. Progressive Multifocal Leukoencephalopathy

Demyelinating ●

D. Charcot Marie Tooth Disease
E. Metachromatic Leukodystrophy

Disease
● F. Central Pontine Myelinolysis
● G. Subacute Sclerosing Panencephalitis
2. Lesions of Vision in Multiple Sclerosis
● A. Optic Neuritis
● B. Marcus Gunn Pupil
● C. Internuclear Ophthalmoplegia
Neurology:
Neurology:Demyelinating Disease
Invasive Spinal Cord Disease Bootcamp.com
Bootcamp.com

• Classic Presentations of GBS


• Recent GI infection or URI
• Ascending symmetric flaccid paralysis (LMN)
• Campylobacter jejuni is the most important to remember overall
• Cytomegalovirus is the most common virus
• Cerebrospinal Fluid Differential
• Albuminocytological dissociation in both Poliomyelitis and GBS
• Pleocytosis more classic for Poliomyelitis
• GBS unlikely with pleocytosis
• Management
• Plasmapheresis or intravenous immunoglobulins
• No corticosteroids
Neurology:
Neurology:Demyelinating Disease
Invasive Spinal Cord Disease Bootcamp.com
Bootcamp.com

• Pathophysiology
• Demyelination in CNS, oligodendrocyte axonal degeneration
• Classic Presentation
• Young patient, classically Caucasian female
• Optic Neuritis à Painful, unilateral
Neurology:
Neurology:Demyelinating Disease
Invasive Spinal Cord Disease Bootcamp.com
Bootcamp.com

• Pathophysiology
• Demyelination in CNS, oligodendrocyte axonal degeneration
• Classic Presentation
• Young patient, classically Caucasian female
• Optic Neuritis à Painful, unilateral
• Marcus Gunn Pupil (relative afferent pupillary defect)
Neurology:
Neurology:Demyelinating Disease
Invasive Spinal Cord Disease Bootcamp.com
Bootcamp.com

• Pathophysiology
• Demyelination in CNS, oligodendrocyte axonal degeneration
• Classic Presentation
• Young patient, classically Caucasian female
• Optic Neuritis à painful, unilateral
• Marcus Gunn Pupil (relative afferent pupillary defect)
• Unilateral/Bilateral Internuclear Ophthalmolpegia (lesion to MLF)
Neurology:
Neurology:Demyelinating Disease
Invasive Spinal Cord Disease Bootcamp.com
Bootcamp.com

• Pathophysiology
• Demyelination in CNS, oligodendrocyte axonal degeneration
• Classic Presentation
• Young patient, classically Caucasian female
• Optic Neuritis à painful, unilateral
• Marcus Gunn Pupil (relative afferent pupillary defect)
• Bilateral Internuclear Ophthalmolpegia (lesion to MLF, heavily myelinated)
• Uhthoff phenomenon
• Bladder or bowel incontinence
• Lhermitte's sign
• Upper motor neuron signs
Neurology:
Neurology:Demyelinating Disease
Invasive Spinal Cord Disease Bootcamp.com
Bootcamp.com

• Imaging: MRI
• Periventricular white matter lesions
• Dawson’s fingers
• Lumbar Puncture
• Oligoclonal bands
• ↑ WBC, predominantly lymphocytes
• Management
• Acute setting consider high dose steroids
• Interferon-!
• Natalizumab à increased risk of progression to PML

https://radiopaedia.org/cases
/multiple-sclerosis-dawsons-
fingers-3
Neurology:
Neurology:Demyelinating Disease
Invasive Spinal Cord Disease Bootcamp.com
Bootcamp.com

• Pathophysiology
• Reactivation of JC virus, typically in immunocompromised host
• Oligodendrocyte destruction à demyelination

• Classic Presentations of PML https://radiopaedia.org/ar


ticles/progressive-

• Poorly controlled HIV history or recent treatment with natalizumab multifocal-


leukoencephalopathy?lan
g=us
• Unusual behavioral changes +/- focal neurologic deficits

• Diagnostics
• Widespread non-enhancing white matter lesions
• JC Virus PCR in CSF

https://en.wikipedia.org/wik
i/Progressive_multifocal_le
ukoencephalopathy
Neurology:
Neurology:Demyelinating Disease
Invasive Spinal Cord Disease Bootcamp.com
Bootcamp.com

• Charcot-Marie-Tooth Disease
• Triad: Hammer toe, pes cavus, foot drop
• Distal, symmetric, sensory and motor neuropathy
• PNS
• Lower motor neuron signs
• Metachromatic Leukodystrophy
• Autosomal recessive, deficiency of arylsulfatase A
• Build up of cerebroside sulfate leading to demyelination
• CNS and PNS
• Motor skill development regress in infants and young children
• Central Pontine Myelinolysis
• Rapid sodium correction
• “Locked in” Syndrome (similar to pontine basilar infarction)
• Subacute Sclerosing Panencephalitis
• History of persistent measles infection
≣ ⟽ ⟾
Item 1 of 1 Test Your Knowledge
◽" Mark Difficulty: ✪✪ Bootcamp.com
Question ID: 0010 Previous Next

A 28-year-old fully vaccinated male with no significant past medical history


⚪ A. Apoptosis of ventral horn cells presents to an urgent care clinic for abdominal cramping and watery
⚪ B. Molecular mimicry and destruction to Schwann cells diarrhea. He is given a short course of antibiotics and his diarrhea
⚪ C. Molecular mimicry and destruction to oligodendrocytes resolves. Three weeks later he presents to the emergency room after a
⚪ D. Replication and reactivation of virus in dorsal root ganglia fall episode. He states he was attempting to walk on the treadmill at his
⚪ E. Inhibition of glycine release via SNARE proteins local fitness center when he lost his balance and fell backward. He states
that over the past week he has had difficulty climbing the stairs and
thought it was because he was sleeping awkwardly on his legs. Muscle
strength testing is 3/5 bilaterally at the lower extremities and reflexes are
absent at S1 bilaterally. A lumbar puncture is performed with findings
shown below. Which of the following is most consistent with the pathologic
mechanism likely causing this patient’s current presentation?

Opening Pressure: 10cm H2O


Appearance: Clear
White Blood Cells: 3 cell/uL (reference range: 0-5 cells/uL)
Protein: 722 mg/L (reference range: <40 mg/dL)
Glucose: 55 mg/dL (reference range: 40-70 mg/dL)
Gram Stain: Negative
No oligoclonal bands are present
≣ ⟽ ⟾
Item 1 of 1 Test Your Knowledge
◽" Mark Difficulty: ✪✪ Bootcamp.com
Question ID: 0010 Previous Next

A 28-year-old fully vaccinated male with no significant past medical history


⚪ A. Apoptosis of ventral horn cells presents to an urgent care clinic for abdominal cramping and watery
$ B. Molecular mimicry and destruction to Schwann cells diarrhea. He is given a short course of antibiotics and his diarrhea
⚪ C. Molecular mimicry and destruction to oligodendrocytes resolves. Three weeks later he presents to the emergency room after a
⚪ D. Replication and reactivation of virus in dorsal root ganglia fall episode. He states he was attempting to walk on the treadmill at his
⚪ E. Inhibition of glycine release via SNARE proteins local fitness center when he lost his balance and fell backward. He states
that over the past week he has had difficulty climbing the stairs and
thought it was because he was sleeping awkwardly on his legs. Muscle
strength testing is 3/5 bilaterally at the lower extremities and reflexes are
absent at S1 bilaterally. A lumbar puncture is performed with findings
shown below. Which of the following is most consistent with the pathologic
mechanism likely causing this patient’s current presentation?

Opening Pressure: 10cm H2O


Appearance: Clear
White Blood Cells: 3 cell/uL (reference range: 0-5 cells/uL)
Protein: 722 mg/L (reference range: <40 mg/dL)
Glucose: 55 mg/dL (reference range: 40-70 mg/dL)
Gram Stain: Negative
No oligoclonal bands are present
OUTLINE

1. Conduction Physiology Concepts

Neurology: ●

A. Extracellular Electrolyte Dynamics
B. Sodium-Potassium-ATPase
● C. Resting Potential

Conduction ●
2.
D. Electrolyte Equilibrium Potential
Action Potential

Physiology ●


A. Stages of the Action Potential
B. Pathology Affecting Current in the Action Potential
C. Saltatory Conduction
● D. Length and Time Constant
Neurology:
Neurology:Conduction Physiology
Invasive Spinal Cord Disease Bootcamp.com
Bootcamp.com

• Electrolyte Physiologic Extracellular Concentrations:


• Sodium (primarily extracellular) ~140 mmol/L
• Potassium (primarily intracellular) : ~5 mmol/L
• Chloride (primarily extracellular) ~105 mmol/L
• Extracellular pH = 7.4
• Negatively charged proteins (primarily intracellular)
Neurology:
Neurology:Conduction Physiology
Invasive Spinal Cord Disease Bootcamp.com
Bootcamp.com

• Resting Potential: membrane potential of a neuron (or muscle cell) at rest


• Sum of all extracellular and intracellular ions
• Ion semi-permeability: Potassium easy transfer, Sodium difficult transfer
• Na+/K+ ATPase: Active transport pump. Hydrolyze 1 ATP, 2 K+ enter cell, 3 Na+ leave cell
• Equilibrium Potential: membrane potential of one ion
• Potassium negative
• Chloride negative
• Sodium positive
Neurology:
Neurology:Conduction Physiology
Invasive Spinal Cord Disease Bootcamp.com
Bootcamp.com

• Action Potential: Voltage-gated induction of a neuronal impulse


• Stage 1: Resting Potential ~ -70-90mV
• Stage 2: Action Potential formed if threshold potential is reached
• Stage 3: Depolarization driven by sodium ion influx
• Stage 4: Repolarization driven by potassium ion efflux
• Stage 5: Afterhyperpolarization

https://commons.wikimedia.org/wiki/File:Acti
on_potential.svg
https://commons.wikimedia.org/wiki/File
:SynapseSchematic_en.svg
Neurology:
Neurology:Conduction Physiology
Invasive Spinal Cord Disease Bootcamp.com
Bootcamp.com

• Inhibitors of Sodium Channels:


• Ciguatoxin à extended depolarization
• Tetrodotoxin à inhibited depolarization
• Lidocaine à inhibited depolarization
• Activators of Chloride Channels:
• GABA and Glycine
• Benzodiazepines à ↑ frequency of Cl- channel opening
• Barbiturates à ↑ duration of Cl- channel opening

https://commons.wikimedia.org/wiki/File:Acti
on_potential.svg
Neurology:
Neurology:Conduction Physiology
Invasive Spinal Cord Disease Bootcamp.com
Bootcamp.com

• Conduction Velocity: Speed of travel of axonal signal


• Continuous Conduction: Unmyelinated nerves
• Action potential travels throughout cell membrane
• Saltatory Conduction: Myelinated nerves
• Nodes of Ranvier
• Faster conduction velocity than similar unmyelinated nerves
• Longer length constant than similar unmyelinated nerves
• Shorter time constant than similar unmyelinated nerves
• Nerve Fiber Diameter: Proportional to conduction velocity
• Axial Resistance: Inversely proportional to conduction velocity
• Membrane Resistance: Proportional to conduction velocity
• Membrane Capacitance: Inversely proportional to conduction velocity
• Length (Space) Constant: Length of travel of axonal signal before decay
• Time Constant: Responsiveness of membrane potential to membrane permeability
≣ ⟽ ⟾
Item 1 of 1 Test Your Knowledge
◽" Mark Difficulty: ✪✪✪✪ Bootcamp.com
Question ID: 0011 Previous Next

A 44-year-old male with a history of major depressive disorder and


Equilibrium Potential generalized anxiety disorder presents to the emergency department in a
lethargic state after his roommate reportedly called paramedics. His
ECl -75 mV respiratory rate is approximately 8 breaths per minute and his oxygen
saturation is 86% on room air. His roommate states that the patient took
EK -90 mV the entire bottle of his prescribed ”anxiety medicine”. Oxygen by nasal
cannula and flumazenil are administered. The patient subsequently
ENa +60 mV becomes more alert, and his respiratory depression improves. Equilibrium
potentials at physiologic conditions are given as a reference.

Which of the following best describes the changes in neuronal conduction


occurring after administration of flumazenil?

⚪ A. Increase in frequency of chloride channel opening


⚪ B. Increase in duration of chloride channel opening
⚪ C. Neuronal potentials will more closely approach chloride equilibrium potentials
⚪ D. Neuronal potentials will more closely approach potassium equilibrium potentials
⚪ E. Neuronal potentials will more closely approach sodium equilibrium potentials
≣ ⟽ ⟾
Item 1 of 1 Test Your Knowledge
◽" Mark Difficulty: ✪✪✪✪ Bootcamp.com
Question ID: 0011 Previous Next

A 44-year-old male with a history of major depressive disorder and


Equilibrium Potential generalized anxiety disorder presents to the emergency department in a
lethargic state after his roommate reportedly called paramedics. His
ECl -75 mV respiratory rate is approximately 8 breaths per minute and his oxygen
saturation is 86% on room air. His roommate states that the patient took
EK -90 mV the entire bottle of his prescribed ”anxiety medicine”. Oxygen by nasal
cannula and flumazenil are administered. The patient subsequently
ENa +60 mV becomes more alert, and his respiratory depression improves. Equilibrium
potentials at physiologic conditions are given as a reference.

Which of the following best describes the changes in neuronal conduction


occurring after administration of flumazenil?

⚪ A. Increase in frequency of chloride channel opening


⚪ B. Increase in duration of chloride channel opening
⚪ C. Neuronal potentials will more closely approach chloride equilibrium potentials
⚪ D. Neuronal potentials will more closely approach potassium equilibrium potentials
$ E. Neuronal potentials will more closely approach sodium equilibrium potentials
OUTLINE

1. Physiology of the Neuromuscular Junction

Neurology: ●

A. Action Potential
B. Synaptic Transmission
● C. Acetylcholinesterase

Neuromuscular 2.

Medications Relevant to the Neuromuscular Junction
A. Antiepileptics

Junction ●

3.
B. Organophosphates
C. Acetylcholinesterase Inhibitors
Diseases Relevant to the Neuromuscular Junction
● A. Myasthenia Gravis
● B. Lambert-Eaton Syndrome
● C. Botulinum Toxin
4. Neuromuscular Blocking Agents
● A. Nondepolarizing
● B. Depolarizing
Neurology:
Neurology:Diseases
InvasiveofSpinal
the Neuromuscular
Cord Disease Junction Bootcamp.com
Bootcamp.com

• Action Potential Reaches Pre-Synaptic Motor End-Plate


• Pre-Synaptic Calcium Channel Opens
• Fusion of ACh Quanta with Neuronal Cell Membrane via SNARE Proteins (calcium required)
• ACh Quanta Released
• ACh Generates Endplate Potentials on Post-Synaptic Membrane
• Ligated-gated sodium channels
• ↑ endplate potential à if threshold potential is reached à action potential generation
• Acetylcholinesterase destroys ACh in synaptic cleft
• Action Potential Triggered Through Sarcolemma
• Voltage-gated sodium channels
Neurology:
Neurology:Diseases
InvasiveofSpinal
the Neuromuscular
Cord Disease Junction Bootcamp.com
Bootcamp.com

• Antiepileptics:
• Phenytoin: Inhibition of voltage gated sodium channels
• Carbamazepine: Inhibition of voltage gated sodium channels
• Gabapentin: Inhibition of presynaptic voltage gated calcium channels
• Levetiracetam: Disrupts vesicle fusion
• Organophosphates:
• Pesticides, Sarin gas: Nonspecific, irreversible inactivation of acetylcholinesterase
• Acetylcholinesterase Inhibitors:
• Edrophonium, Physostigmine, Pyridostigmine: Inhibits acetylcholinesterase
Neurology:
Neurology:Diseases
InvasiveofSpinal
the Neuromuscular
Cord Disease Junction Bootcamp.com
Bootcamp.com

• Pathophysiology: Anti-acetylcholine receptor antibodies (most classic)


• Binds to post-synaptic nicotinic acetylcholine receptors à inhibition of muscle activation
• Presentation: Fatigable muscle weakness, worse with activity, improves with rest
• Extraocular à Ptosis, diplopia
• Bulbar à Dysphagia, dysarthria
• Respiratory à Dyspnea
• Limb weakness à proximal muscles
• Reflexes tend to be normal
• Diagnostics: AChR Antibodies, Chest CT, EMG
• Other: Ice pack test, Edrophonium test
• Associated Pathology: Thymoma
• Associated Diseases: Hashimoto thyroiditis, SLE, RA
• Management: Thymectomy, Pyridostigmine, Glucocorticoids https://commons.wikime
• Myasthenic Crisis: Trigger à severe exacerbation à respiratory failure, Rx: IVIG, dia.org/wiki/File:Anterior
_mediastinal_mass_thy
plasmapheresis moma_diagram.jpg
Neurology:
Neurology:Diseases
InvasiveofSpinal
the Neuromuscular
Cord Disease Junction Bootcamp.com
Bootcamp.com

• Pathophysiology: Anti-voltage-gated calcium channels antibodies (most classic)


• Binds to pre-synaptic voltage-gated calcium channels à ↓ acetylcholine release
• Presentation: Improving muscle weakness with activity
• Limb weakness à proximal muscles
• Autonomic symptoms à constipation, dry oral mucosa, orthostatic hypotension
• Reflexes tend to be reduced or absent
• Diagnostics: Anti-VGCC Antibodies, Chest CT, EMG
• Associated Pathology: Small cell lung cancer
• Associated Diseases: Hashimoto thyroiditis, SLE, RA
• Management: Amifampridine
Neurology:
Neurology:Diseases
InvasiveofSpinal
the Neuromuscular
Cord Disease Junction Bootcamp.com
Bootcamp.com

• Pathophysiology: Cleaves SNARE proteins (most classic)


• Prevents fusion of neurotransmitter at pre-synaptic cleft à ↓ acetylcholine release
• Pathogen: Clostridium botulinum, gram + rod, obligate anaerobe, spore forming
• Botulinum toxin produced by Clostridium botulinum
• Presentation: Descending flaccid paralysis
• Ocular à Dilated pupil, diplopia
• Bulbar à Dysphagia, dysarthria, facial muscle weakness
• Autonomic symptoms à dry oral mucosa
• Respiratory à Dyspnea
• Foodborne: Preformed toxin in food, classically canned foods, raw honey and juice (infants)
• Infant Botulism: Floppy baby syndrome (also other causes)
• Wound Botulism: Preformed toxin into wound, long incubation period, classic IVDU
• Management: Always airway first à Immune globulin (equine vs human)
• Management: Amifampridine
Neurology:
Neurology:Diseases
InvasiveofSpinal
the Neuromuscular
Cord Disease Junction Bootcamp.com
Bootcamp.com

• Nondepolarizing: Competitive antagonists of post-synaptic nicotinic acetylcholine receptors


• Vecuronium, Rocuronium
• Depolarizing: Competitive agonist of post-synaptic nicotinic acetylcholine receptors
• Succinylcholine
Neurology:
Neurology:Diseases
InvasiveofSpinal
the Neuromuscular
Cord Disease Junction Bootcamp.com
Bootcamp.com

Neuromuscular Junction Pathology


Disease Process Pathophysiology Hallmark Findings Reflexes Association

Myasthenia Gravis Anti-AChR Abs (usually) (post- Fatigable muscle Normal Thymoma
synaptic) weakness

Lambert-Eaton Syndrome Anti-VGCC Abs (pre-synaptic) Improving muscle Decreased/absent Small cell lung
weakness with activity cancer

Botulism Protease cleaving SNARE Descending flaccid Variable Canned foods


preventing ACh release (pre- paralysis and honey
synaptic) (infants)

Guillian-Barre Syndrome Molecular mimicry to PNS Ascending symmetric Decreased/absent Previous illness
flaccid paralysis
≣ ⟽ ⟾
Item 1 of 1 Test Your Knowledge
◽" Mark Difficulty: ✪✪✪ Bootcamp.com
Question ID: 0012 Previous Next

A 37-year-old-male with a past medical history of asthma comes into the


⚪ A. Significantly lower than standard dosing is optimal emergency room stating that he is having trouble breathing. He states that
⚪ B. Slightly lower than standard dosing is optimal he went for a walk outside and became very weak and short of breath. He
⚪ C. Standard dosing is optimal also states that he over the past year he has noticed worsening double
⚪ D. Slightly greater than standard dosing is optimal vision toward the end of the day, but it seems to resolve after sleeping.
⚪ E. Significantly greater than standard dosing is optimal He also reports difficulty chewing meats, dysphagia, and has since
decided to become a vegetarian. An image of his eyes is depicted below.

Prior to any breathing treatment, the patient’s respiratory distress acutely


worsens. Succinylcholine is given for muscle relaxation in addition to
general anesthesia to facilitate endotracheal intubation.

Which of the following will most likely be true regarding the optimal dose
of succinylcholine for this patient?

https://commons.wikimedia.org/wiki/File:Congenitalptosis.JPG
≣ ⟽ ⟾
Item 1 of 1 Test Your Knowledge
◽" Mark Difficulty: ✪✪✪ Bootcamp.com
Question ID: 0012 Previous Next

A 37-year-old-male with a past medical history of asthma comes into the


⚪ A. Significantly lower than standard dosing is optimal emergency room stating that he is having trouble breathing. He states that
⚪ B. Slightly lower than standard dosing is optimal he went for a walk outside and became very weak and short of breath. He
⚪ C. Standard dosing is optimal also states that he over the past year he has noticed worsening double
⚪ D. Slightly greater than standard dosing is optimal vision toward the end of the day, but it seems to resolve after sleeping. He
$ E. Significantly greater than standard dosing is optimal also reports difficulty chewing meats, dysphagia, and has since decided to
become a vegetarian. An image of his eyes is depicted below.

Prior to any breathing treatment, the patient’s respiratory distress acutely


worsens. Succinylcholine is given for muscle relaxation in addition to
general anesthesia to facilitate endotracheal intubation.

Which of the following will most likely be true regarding the optimal dose
of succinylcholine for this patient?

https://commons.wikimedia.org/wiki/File:Congenitalptosis.JPG
OUTLINE

1. Cranial Nerves

Neurology: ●

A. Olfactory Nerve
B. Optic Nerve
● C. Oculomotor Nerve

Cranial Nerves I-VI ●



D. Trochlear Nerve
E. Trigeminal Nerve
● F. Abducens Nerve
2. High Yield Considerations
● A. Compression vs Ischemic Lesions of Oculomotor Nerve
● B. Cavernous Sinus
Neurology: Cranial Nerves I-VI Bootcamp.com

• Cranial Nerve I (Olfactory Nerve)


• Not located in brainstem
• No thalamic relay
• Sensory nerve
• Etiology of Lesions
• Ethmoid bone fracture (cribriform plate)
• Meningioma
• Presentation
• Anosmia
Neurology: Cranial Nerves I-VI Bootcamp.com

• Cranial Nerve II (Optic Nerve)


• Not located in brainstem
• Sensory nerve responsible for vision
• Afferent limb of the pupillary light reflex
• Travels through optic canal
• Etiology of Lesions
• Multiple sclerosis
• Microvascular disease
• Ethambutol
• Presentation
• Optic neuritis à loss of vision with painful eye movements
• Marcus Gunn pupil à swinging flashlight test
• Loss of ipsilateral and consensual light reflex when affected nerve is tested
• Anopsia
Neurology: Cranial Nerves I-VI Bootcamp.com

• Cranial Nerve III (Oculomotor Nerve)


• Nucleus in midline midbrain
• Motor sensation to superior, inferior, and medial rectus, inferior oblique, and levator palpebrae superioris
• Efferent limb of light reflex
• Parasympathetic fibers to pupillary sphincter and ciliary muscles
• Travels through the superior orbital fissure
• Etiology of Lesions
• Posterior communicating artery aneurysm
• Multiple sclerosis (specifically affecting the MLF, CN III downstream)
• Uncal herniation
• Microvascular disease
• Presentation
• Ptosis, pupillary dilation +/- anhidrosis if Horner Syndrome
• Internuclear ophthalmoplegia
• Loss of ipsilateral light reflex when affected nerve is tested
• Loss of consensual light reflex when unaffected nerve is tested
• ”Down and Out” Gaze; Impaired adduction; Diagonal diplopia
Neurology: Cranial Nerves I-VI Bootcamp.com

• Oculomotor Nerve Anatomical Considerations


• Parasympathetic fibers running superficial to motor fibers
• Ischemic and/or Demyelinating Lesions
• Motor >= Parasympathetic dysfunction
• Microvascular disease (diabetes)
• Ocular vasculitis
• Demyelinating injury (to MLF for example)
• Compressive Lesions
• Motor < Parasympathetic dysfunction
• Posterior communicating artery aneurysm
• Uncal herniation
Neurology: Cranial Nerves I-VI Bootcamp.com

• Cranial Nerve IV (Trochlear Nerve)


• Nucleus midline midbrain
• Motor sensation to superior oblique muscle
• Travels through superior orbital fissure
• Etiology
• Microvascular disease
• Presentation
• Inability to depress and adduct the eye
• Worsened when looking down away from the affected side
• Head tilt to the opposite side
• Vertical and torsional diplopia
Neurology: Cranial Nerves I-VI Bootcamp.com

• Cranial Nerve V (Trigeminal Nerve)


• Nucleus lateral pons
• Sensory to face: Three divisions (V1, V2, V3)
• Sensory to anterior 2/3 of the tongue (V3)
• Motor sensation to muscles of mastication
• Afferent and efferent limb of jaw jerk reflex (V3)
• Afferent corneal and lacrimation reflexes (V1)
• Cranial nerve V1 travels through superior orbital fissure
• Cranial nerve V2 travels through foramen rotundum and inferior orbital fissure
• Cranial nerve V3 travels through foramen ovale
• Etiology
• Foraminal Lesions
• Trigeminal Neuralgia
• Temporomandibular Joint (TMJ) Dysfunction
• Herpes Zoster Ophthalamicus
• Presentation
• Lesion to V1 à Sensation in V1, loss of corneal and lacrimation reflexes
• Lesion to V2à Sensation in V2
• Lesion to V3à Sensation in V3, anterior 2/3 of the tongue, contralateral paralysis in muscles of mastication (Jaw deviates ipsilateral to lesion)

https://commons.wikimedia.org/wiki/File:Trig_innervation.svg
Neurology: Cranial Nerves I-VI Bootcamp.com

• Cranial Nerve VI (Abducens Nerve)


• Nucleus midline pons
• Motor sensation to lateral rectus muscle
• Etiology
• Elevations in intra-cranial pressure (herniation, hemorrhage, hematoma, pseudo-tumor cerebri)
• Presentation
• Inability to perform abduction of affected eye
• Horizontal diplopia
Neurology: Cranial Nerves I-VI Bootcamp.com

• Etiology
• Infectious cause à retrograde through valveless venous system
• Hypercoagulable states
• Presentation
• Signs of increased intracranial pressure (bilateral papilledema, headaches)
• Fever
• Painful eye movements
• Degree of paralysis with eye movements
• Loss of facial sensation
• Absent corneal reflex (afferent limb)

https://upload.wikimedia.org/wikipedia/commons/b/bf/Gray571.png https://commons.wikimedia.org/wiki/File:Papilledema.jpg
≣ ⟽ ⟾
Item 1 of 1 Test Your Knowledge
◽" Mark Difficulty: ✪✪✪ Bootcamp.com
Question ID: 0013 Previous Next

A 76-year-old male current everyday smoker is being evaluated in by his


primary care physician for an “enlarging pimple” on his nose that he first
noticed 3 days ago. He states that prior to noticing the lesion, he had
been experiencing severe stinging pain on the skin. He states that the
pain has been localized to the left side of the tip of the nose. He recently
started a short course of steroids for an exacerbation of his underlying
chronic bronchitis and emphysema. He denies any shortness of breath
A today. On physical exam, the lesion appears to be moderately tender to
touch and vesicular in nature. Small blisters are formed centrally in the
lesion draining minor quantities of serous exudate. Valacyclovir is initiated
B and the patient is instructed to return immediately if the rash is worsening
or if he experiences any fever or vision changes.

C Which of the following is most likely traversed by the primary nerve


involved in this patient’s presentation?
D ⚪ A.
⚪ B.
⚪ C.
⚪ D.
E ⚪ E.

https://commons.wikimedia.org/wiki/File:Skull_foramina_labeled.svg
≣ ⟽ ⟾
Item 1 of 1 Test Your Knowledge
◽" Mark Difficulty: ✪✪✪ Bootcamp.com
Question ID: 0013 Previous Next

A 76-year-old male current everyday smoker is being evaluated in by his


primary care physician for an “enlarging pimple” on his nose that he first
noticed 3 days ago. He states that prior to noticing the lesion, he had
been experiencing severe stinging pain on the skin. He states that the
pain has been localized to the left side of the tip of the nose. He recently
started a short course of steroids for an exacerbation of his underlying
chronic bronchitis and emphysema. He denies any shortness of breath
A today. On physical exam, the lesion appears to be moderately tender to
touch and vesicular in nature. Small blisters are formed centrally in the
lesion draining minor quantities of serous exudate. Valacyclovir is initiated
B and the patient is instructed to return immediately if the rash is worsening
or if he experiences any fever or vision changes.

C Which of the following is most likely traversed by the primary nerve


involved in this patient’s presentation?
D ⚪ A.
$ B.
⚪ C.
⚪ D.
E ⚪ E.

https://commons.wikimedia.org/wiki/File:Skull_foramina_labeled.svg
OUTLINE

1. Cranial Nerves

Neurology: ●

A. Facial Nerve
B. Vestibulocochlear Nerve
● C. Glossopharyngeal Nerve

Cranial Nerves ●

D. Vagus Nerve
E. Spinal Accessory Nerve

VII-XII
● F. Hypoglossal Nerve
2. High Yield Considerations
● A. Facial Nerve Anatomy
Neurology: Cranial Nerves VII-XII Bootcamp.com

• Cranial Nerve VII (Facial Nerve)


• Nucleus in lateral pons
• Taste anterior 2/3 of tongue (chorda tympani branch)
• Motor to facial muscles
• Efferent limb of corneal and lacrimation reflex
• Parasympathetic control over lacrimation and salivation
• Afferent somatic fibers from the pinna and external auditory canal
• Etiology of Lesions
• Parotid gland mass (sialadenitis, pleomorphic adenoma, mucoepidermoid carcinoma)
• Bell’s Palsy (Idiopathic Facial Nerve Palsy)
• Temporal bone trauma
• Presentation
• Loss of taste sensation over anterior 2/3 of tongue
• Impaired eye closure, drooping of face, impaired ability to smile
• Loss of corneal reflex (efferent)
• Disappearance of nasolabial fold
• Impaired lacrimation https://commons.wikimedia.org/wiki/File:Parotid_gland_en.png
• Hyperacusis (stapedius)
Neurology: Cranial Nerves VII-XII Bootcamp.com

• Central Lesion (ex. CVA)


• Upper Facial Muscles: Dual innervation, remain intact
• Lower Facial Muscles: Dysfunction contralateral to the lesion
• Peripheral Lesion (ex. Bell’s Palsy)
• Upper Facial Muscles: Dysfunction ipsilateral to the lesion
• Lower Facial Muscles: Dysfunction ipsilateral to the lesion

https://commons.wikimedia.org/wiki/File:Cranial_nerve_VII.svg
Neurology: Cranial Nerves VII-XII Bootcamp.com

• Cranial Nerve VIII (Vestibulocochlear Nerve)


• Nucleus in lateral pons and medulla
• Sensory modalities in balance, auditory, and equilibrium
• Etiology of Lesions
• Bilateral Acoustic Neuromas: Neurofibromatosis Type 2
• Basilar Skull Fracture
• Temporal bone trauma
• Presentation
• Sensorineural hearing loss
• Vertigo
• Balance Impairment
• Horizontal Nystagmus
• Basilar Skull Fracture à ”Raccoon eyes” (periorbital ecchymoses)
https://commons.wikimedia.org/wiki/File:Bilateral_periorbital_ecchymosis_(raccoon_eyes).jpg
https://www.researchgate.net/figure/Vestibular-schwannomas-Axial-A-and-coronal-B-enhanced-
T1-weighted-MR-images_fig1_38012259
Neurology: Cranial Nerves VII-XII Bootcamp.com

• Cranial Nerve IX (Glossopharyngeal Nerve)


• Nucleus in lateral medulla
• Sensory innervation and taste to posterior 1/3 of tongue
• Sensory innervation to eustachian tube, upper pharynx, and inner surface of tympanic membrane
• Motor to pharyngeal muscles à swallowing (Stylopharyngeus)
• Afferent limb of gag reflex
• Baroreceptors and Chemoreceptors
• Parasympathetics for salivation (otic ganglion à auriculotemporal n.à parotid gland )
• Etiology of Lesions
• Iatrogenic
• Presentation
• Loss of gag reflex
• Dysphagia
• Loss of taste sensation in posterior 1/3 of tongue
• Sensory loss in upper pharynx, posterior tongue, middle ear cavity
Neurology: Cranial Nerves VII-XII Bootcamp.com

• Cranial Nerve X (Vagus Nerve)


• Nucleus in lateral medulla
• Sensory innervation distal to posterior tongue
• Motor to pharyngeal muscles à swallowing and speech
• Efferent limb of gag reflex
• Baroreceptors and Chemoreceptors
• Parasympathetics for visceral organ and vessel function
• Etiology of Lesions
• Recurrent Laryngeal Nerve à Thyroid tumor, apical lung tumor
• Presentation
• Hoarseness, dysphagia, dysphonia
• Uvula deviated away from the side of the lesion
• Impaired gag reflex

https://commons.wikimedia.org/wiki/File:Recurrent_laryngeal_nerve.svg
Neurology: Cranial Nerves VII-XII Bootcamp.com

• Cranial Nerve XI (Spinal Accessory Nerve)


• Nucleus in lateral medulla
• Motor to SCM and trapezius muscles
• Etiology of Lesions
• Iatrogenicà Cervical lymph node dissection
• Presentation
• Weakness with head turning to the contralateral side
• Ipsilateral shoulder drooping

https://commons.wikimedia.org/wiki/File:Gray793.png
Neurology: Cranial Nerves VII-XII Bootcamp.com

• Cranial Nerve XII (Hypoglossal Nerve)


• Nucleus in midline medulla
• Motor to intrinsic and extrinsic tongue muscles except palatoglossus
• Etiology of Lesions
• Internal carotid artery dissection
• Iatrogenic
• Presentation
• Supranuclear injury à tongue deviation away from side of lesion, UMN signs
• Hypoglossal nerve injury à tongue deviation toward side of lesion, LMN signs
≣ ⟽ ⟾
Item 1 of 1 Test Your Knowledge
◽" Mark Difficulty: ✪✪ Bootcamp.com
Question ID: 0014 Previous Next

A 54-year-old female with a history of medullary carcinoma of the thyroid


presents to her local urgent care clinic with complaints of difficulty
speaking. She states that she is employed as a marketing manager and is
having difficulty completing presentations. Her voice seems to “disappear”
after thirty minutes of talking. She reports she has been consuming more
water than usual to try to “clear her throat”. She denies any globus
sensation, cough, or recent infection. On physical exam the lungs are
clear to auscultation bilaterally and the heart has a regular rate and
rhythm. The uvula and tongue are noted to be midline with no asymmetry
in soft palate elevation. No focal neurologic deficits are noted in the
extremities, and she has appropriate muscle strength with shoulder
shrugging. She reports recently undergoing a total thyroidectomy and
neck dissection prior to her first noticing these symptoms.

Which of the following lesions are most consistent with this patient’s
presentation?
⚪ A. Metastatic spread of primary tumor to the hypoglossal canal
⚪ B. Metastatic spread of primary tumor to the jugular foramen
⚪ C. Iatrogenic injury in proximity to distal inferior thyroid artery
⚪ D. Iatrogenic hypoperfusion injury at the dorsal motor nucleus
⚪ E. Metastatic spread of primary tumor to the foramen magnum
≣ ⟽ ⟾
Item 1 of 1 Test Your Knowledge
◽" Mark Difficulty: ✪✪ Bootcamp.com
Question ID: 0014 Previous Next

A 54-year-old female with a history of medullary carcinoma of the thyroid


presents to her local urgent care clinic with complaints of difficulty
speaking. She states that she is employed as a marketing manager and is
having difficulty completing presentations. Her voice seems to “disappear”
after thirty minutes of talking. She reports she has been consuming more
water than usual to try to “clear her throat”. She denies any globus
sensation, cough, or recent infection. On physical exam the lungs are
clear to auscultation bilaterally and the heart has a regular rate and
rhythm. The uvula and tongue are noted to be midline with no asymmetry
in soft palate elevation. No focal neurologic deficits are noted in the
extremities, and she has appropriate muscle strength with shoulder
shrugging. She reports recently undergoing a total thyroidectomy and
neck dissection prior to her first noticing these symptoms.

Which of the following lesions are most consistent with this patient’s
presentation?

⚪ A. Metastatic spread of primary tumor to the hypoglossal canal


⚪ B. Metastatic spread of primary tumor to the jugular foramen
$ C. Iatrogenic injury in proximity to distal inferior thyroid artery
⚪ D. Iatrogenic hypoperfusion injury at the dorsal motor nucleus
⚪ E. Metastatic spread of primary tumor to the foramen magnum
OUTLINE

1. Visual Pathway

Neurology: ●

A. Retina and Optic Nerve
B. Optic Chiasm
● C. Peri-chiasm

Vision ●

D. Lateral Geniculate Nucleus
E. Optic Radiations
● F. Primary Visual Cortex
2. Pupillary Light Reflex
● A. Afferent Pupillary Defect
● B. Efferent Pupillary Defect
Neurology: Vision Bootcamp.com

• Visual Pathway:
• Retina à optic n. à optic chiasm à optic tract à LGN à optic radiations à visual cortex
• Pupillary Light Reflex Pathway:
• Retina à optic n. à optic chiasm à optic tract à pretectal area à EWP nucleus à CN III
• Terminology:
• Temporal visual field à projects onto nasal retina
• Nasal visual field à projects onto temporal retina
• Classifying Lesions:
• Anopia
• Hemianopia
• Quadrantanopia
• Homonymous
• Nasal/Temporal
Neurology: Vision Bootcamp.com

• Structures Involved: Retina, optic nerve


• Visual field and retinal fibers are inversely related
• General Presentation:
• Ipsilateral scotoma (partial)
• Ipsilateral anopia (complete)
• Loss of afferent limb of pupillary light reflex
• Etiology of Lesions:
• Macular degeneration
• Central retinal artery occlusion
• Central retinal vein occlusion
• Optic neuritis (multiple sclerosis)
• Ethambutol
• Optic atrophy
• Diabetic retinopathy
• Retinitis pigmentosa https://en.wikipedia.org/wiki/Fundus_(eye
)#/media/File:Fundus_photograph_of_nor
https://commons.wikimedia.org/wiki/File:Cherry_red_spot_in_pati
ent_with_central_retinal_artery_occlusion_(CRAO).jpg
mal_right_eye.jpg
Neurology: Vision Bootcamp.com

• Structures Involved: Optic Chiasm


• Contains temporal visual field / nasal retinal fibers
• General Presentation:
• Bitemporal heteronymous hemianopia
• Etiology of Lesions:
• Pituitary lesion (prolactinoma)
• Craniopharyngioma
• Anterior communicating artery aneurysm
Neurology: Vision Bootcamp.com

• Structures Involved: Peri-chiasm


• Contains nasal visual field / temporal retinal fibers
• General Presentation:
• Ipsilateral nasal hemianopia
• Etiology of Lesions:
• Internal carotid artery aneurysms
• Internal carotid artery calcification
Neurology: Vision Bootcamp.com

• Structures Involved: Optic tract, lateral geniculate nucleus


• Contains ipsilateral nasal visual field fibers and contralateral temporal visual field fibers
• General Presentation:
• Contralateral homonymous hemianopia
• Etiology of Lesions:
• Middle cerebral artery infarction
Neurology: Vision Bootcamp.com

• Structures Involved: Optic radiations


• Meyer’s Loop (Temporal lobe): Inferior bundle containing ipsilateral superior nasal visual field fibers and contralateral
superior temporal visual field fibers
• Superior Loop (Parietal lobe): Superior bundle containing ipsilateral inferior nasal visual field fibers and contralateral
inferior temporal visual field fibers
• General Presentation:
• Meyer’s Loop: Contralateral superior homonymous quadrantanopia
• Superior Loop: Contralateral inferior homonymous quadrantanopia
• Etiology of Lesions:
• Middle cerebral artery infarction
• Lesion to posterior limb of internal capsule
Neurology: Vision Bootcamp.com

• Structures Involved: Visual Cortex


• Contains projections of ipsilateral nasal and contralateral temporal visual field fibers
• Occipital Lobe
• General Presentation:
• Contralateral homonymous hemianopia with macular sparing
• Etiology of Lesions:
• Posterior cerebral artery infarction
≣ ⟽ ⟾
Item 1 of 1 Test Your Knowledge
◽" Mark Difficulty: ✪✪✪ Bootcamp.com
Question ID: 0015 Previous Next

A 67-year-old male with a past medical history of type 2 diabetes mellitus


presents to the emergency department for color blindness that “comes
and goes”. He states that everything appears to have a transient blue-like
discoloration. He denies any loss of vision but reports after staring at
certain objects his vision tends to become increasingly blurry. His vital
signs are notable for a blood pressure of 92/58. He states that his blood
pressure was borderline high about one year ago but has since been
improved since he started taking sildenafil for his erectile dysfunction. His
visual acuity is normal and there are no deficits in the visual fields. The
risks and benefits to sildenafil use are discussed with the patient and he is
subsequently discharged. One week later he returns to the emergency in
a panic with complete loss of vision in the right eye. His extraocular
movements are intact with no other apparent focal neurologic deficits.

Which of the following findings would most likely be consistent with this
patient’s presentation?

⚪ A. Significant miosis bilaterally prior to pupillary light reflex testing


⚪ B. Failure to obtain a direct and consensual response when light is shone in the left eye
⚪ C. Failure to obtain a direct response when light is shone in the left eye; consensual response without deficits
⚪ D. Failure to obtain a direct response when light is shone in the right eye; consensual response without deficits
⚪ E. Appropriately normal direct and consensual response when light is shone in the left eye
≣ ⟽ ⟾
Item 1 of 1 Test Your Knowledge
◽" Mark Difficulty: ✪✪✪ Bootcamp.com
Question ID: 0015 Previous Next

A 67-year-old male with a past medical history of type 2 diabetes mellitus


presents to the emergency department for color blindness that “comes
and goes”. He states that everything appears to have a transient blue-like
discoloration. He denies any loss of vision but reports after staring at
certain objects his vision tends to become increasingly blurry. His vital
signs are notable for a blood pressure of 92/58. He states that his blood
pressure was borderline high about one year ago but has since been
improved since he started taking sildenafil for his erectile dysfunction. His
visual acuity is normal and there are no deficits in the visual fields. The
risks and benefits to sildenafil use are discussed with the patient and he is
subsequently discharged. One week later he returns to the emergency in
a panic with complete loss of vision in the right eye. His extraocular
movements are intact with no other apparent focal neurologic deficits.

Which of the following findings would most likely be consistent with this
patient’s presentation?

⚪ A. Significant miosis bilaterally prior to pupillary light reflex testing


⚪ B. Failure to obtain a direct and consensual response when light is shone in the left eye
⚪ C. Failure to obtain a direct response when light is shone in the left eye; consensual response without deficits
⚪ D. Failure to obtain a direct response when light is shone in the right eye; consensual response without deficits
$ E. Appropriately normal direct and consensual response when light is shone in the left eye
OUTLINE

1. Fundamental Concepts

Neurology: ●

A. Basic Ear Anatomy
B. Stapedius, Tensor Tympani
● C. Acoustic Reflex

Auditory 2.

Clinical Diagnostics for Hearing Loss
A. Weber Test
● B. Rinne Test
● C. Conductive Hearing Loss
● D. Sensorineural Hearing Loss
3. Causes of Conductive Hearing Loss
● A. Cerumen Impaction
● B. Otitis Media
● C. Otosclerosis
4. Causes of Sensorineural Hearing Loss
● A. Acquired Noise Hearing Impairment
● B. Vestibular Schwannoma
● C. Presbycusis
● D. Meniere Disease
Neurology: Auditory Bootcamp.com

• Auditory Ossicles:
• Malleus
• Incus
• Stapes
• Skeletal Muscles:
• Tensor tympani: mandibular branch of trigeminal nerve
• Stapedius: facial nerve à stapedius nerve
• Acoustic Reflex:
• Contraction of tensor tympani and stapedius in response to loud noises

https://commons.wikimedia.org/wiki/File:Anatomy_of_the_Human_Ear.svg
Neurology: Vision Bootcamp.com

• Structures Involved: Visual Cortex


• Contains projections of ipsilateral nasal and contralateral temporal visual field fibers
• Occipital Lobe
• General Presentation:
• Contralateral homonymous hemianopia with macular sparing
• Etiology of Lesions:
• Posterior cerebral artery infarction
Neurology: Auditory Bootcamp.com

• Weber Test
• Normal: vibration equal bilaterally
• Abnormal: vibration asymmetric (lateralization)
• Rinne Test
• Normal: air conduction > bone conduction*
• Conductive Hearing Loss
• External or middle ear
• Lateralizes to affected ear
• Bone conduction > air conduction in affected ear
• Sensorineural Hearing Loss
• Inner ear, cochlear nerve
• Lateralizes to unaffected ear
• Air conduction > bone conduction in both ears
Neurology: Auditory Bootcamp.com

Weber and Rinne Tests


Weber Rinne Rinne
Left Ear Right Ear

Normal (no lateralization) AC > BC AC > BC Normal hearing


Bilateral sensorineural hearing loss

Normal (no lateralization) BC > AC BC > AC Bilateral conductive hearing loss


(symmetric)

Right lateralization AC > BC BC > AC Right conductive hearing loss

Left lateralization BC > AC AC > BC Left conductive hearing loss

Right lateralization AC > BC AC > BC Left sensorineural hearing loss

Left lateralization AC > BC AC > BC Right sensorineural hearing loss


Neurology: Auditory Bootcamp.com

• Cerumen Impaction
• External auditory canal, conductive
• Otitis Media
• Infection of middle ear cavity, conductive
• Otosclerosis
• Stapes fixed against oval window, conductive
• Obstructing Mass in External Auditory Canal:
• Conductive
Neurology: Auditory Bootcamp.com

• Mechanical Sound Transduction Into Nerve Impulse (organ of Corti)


• Tympanic membrane à oval window à basilar membrane à
• Hair cell cilia move against tectorial membrane
• Depolarization and hyperpolarization through cochlear nerve fibers
• Acquired Noise-Induced Hearing Impairment
• Prolonged damage to stereociliated hair cells
• High frequency, bilateral, sensorineural
• Vestibular Schwannoma
• Trigeminal, facial, and vestibulocochlear nerve in proximity
• Cerebellopontine angle
• Sensorineural, usually unilateral (unless NF Type 2)
• Presbycusis
• Older age, high frequency, bilateral, sensorineural
• Meniere Disease
• Low frequency tinnitus, sensorineural
https://commons.wikimedia.org/wiki/File:Akustikusneurinom_Mrt.jpg https://commons.wikimedia.org/wiki/File:Organ_of_corti.svg
≣ ⟽ ⟾
Item 1 of 1 Test Your Knowledge
◽" Mark Difficulty: ✪✪✪ Bootcamp.com
Question ID: 0016 Previous Next

A 24-year-old male presents to his family medicine physician for foul-


smelling drainage from the left ear. He also states that he has
experienced worsening hearing loss from his left side for the past year.
He states that he works as a librarian and rarely listens to music. He
denies any loss of sensation on the face or fever. A fundoscopic exam is
performed revealing a gray-brown irregularly shaped mass with dark
discoloration to the surrounding epithelia. Scant amounts of discharge are
noted.

Which of the following results of the Weber and Rinne test would be most
consistent with the finding in this patient?

Legend:
Air conduction: AC
⚪ A. Left AC > BC, Right BC > AC, lateralization to the right Bone conduction: BC
⚪ B. Left AC > BC, Right AC > BC, lateralization to the left
⚪ C. Left BC > AC, Right AC > BC, lateralization to the left
⚪ D. Left AC > BC, Right AC > BC, lateralization to the right
⚪ E. Left AC > BC, Right AC > BC, no lateralization

https://commons.wikimedia.org/wiki/File:Cholesteatom_kuppelraum_1a.jpg
≣ ⟽ ⟾
Item 1 of 1 Test Your Knowledge
◽" Mark Difficulty: ✪✪✪ Bootcamp.com
Question ID: 0016 Previous Next

A 24-year-old male presents to his family medicine physician for foul-


smelling drainage from the left ear. He also states that he has
experienced worsening hearing loss from his left side for the past year.
He states that he works as a librarian and rarely listens to music. He
denies any loss of sensation on the face or fever. A fundoscopic exam is
performed revealing a gray-brown irregularly shaped mass with dark
discoloration to the surrounding epithelia. Scant amounts of discharge are
noted.

Which of the following results of the Weber and Rinne test would be most
consistent with the finding in this patient?

Legend:
Air conduction: AC
⚪ A. Left AC > BC, Right BC > AC, lateralization to the right Bone conduction: BC
⚪ B. Left AC > BC, Right AC > BC, lateralization to the left
$ C. Left BC > AC, Right AC > BC, lateralization to the left
⚪ D. Left AC > BC, Right AC > BC, lateralization to the right
⚪ E. Left AC > BC, Right AC > BC, no lateralization

https://commons.wikimedia.org/wiki/File:Cholesteatom_kuppelraum_1a.jpg
OUTLINE

1. Fundamental Concepts

Neurology: ●

A. Vestibular System Anatomy
B. Peripheral vs Central Vertigo
2. Peripheral Vertigo
Vertigo ●

A. Benign Paroxysmal Positional Vertigo
B. Vestibular Neuritis, Labyrinthitis
● C. Meniere’s Disease, Acoustic Neuroma
3. Central Vertigo
● A. Cerebellar Infarction
● B. Brainstem Lesions
● C. Brain Tumor (Posterior Fossa)
● D. Multiple Sclerosis
Neurology: Vertigo Bootcamp.com

• Vestibular System
• Semicircular canals: detect angular acceleration in 3 planes
• Saccule: detect linear motion in vertical plane
• Utricle: detect linear motion in horizontal plane
Neurology: Vertigo Bootcamp.com

• Peripheral Vertigo
• Lesion à inner ear or vestibulocochlear nerve Vertigo
• Vertigo ceases or improves with visual fixation
• Positional
• Motor, gait, coordination generally intact Peripheral Central
• Central Vertigo
• Lesion à Cerebellum, brainstem nuclei
• Vertigo does not cease with visual fixation
• Non-positional, generally continuous
• Motor, gait, coordination impairment may be present
Neurology: Vertigo Bootcamp.com

Peripheral
• Pathophysiology:
• Dislodged endolymphatic debris (otoconia) à disrupted semicircular canal function
• Presentation:
• Paroxysmal à sudden onset, lasting less than 1 minute
• Positional à precipitated by specific head movements
• Nystagmus
• Diagnostics and Management:
• Clinical diagnosis
• Dix-Hallpike maneuver à if + test à Epley maneuver
Neurology: Vertigo Bootcamp.com

• Pathophysiology:
• Inflammatory reaction of vestibular nerve Peripheral
• Presentation:
• Persistent à degree of vertigo present continuously
• Positional à precipitated by specific head movements
• Previous viral URI or otitis media
• Nystagmus
• Labyrinthitis: Similar presentation + hearing loss, possibly tinnitus
• Diagnostics and Management:
• Clinical diagnosis
Neurology: Vertigo Bootcamp.com

Peripheral
• Pathophysiology:
• Impaired resorption of endolymph à ↑ volume of endolymph
• Presentation:
• Episodic, no trigger (usually) lasts for minutes to hours
• Unilateral (usually)
• Tinnitus
• Sensorineural hearing loss (normal Rinne test, lateralization of Weber test to unaffected)
• Nystagmus
• Acoustic Neuroma: Similar presentation, but not episodic
• Diagnostics and Management:
• Acute attack à Vestibular suppressants (meclizine)
• Secondary prevention à Low sodium diet, avoid triggers
• Chronic management (refractory) à Thiazide diuretics
Neurology: Vertigo Bootcamp.com

Central
• Cerebellar Infarction:
• Brain tumor (posterior fossa):
• Adults: Ependymoma, meningioma, metastatic disease
• Children: Medulloblastomas
• Central Nervous System Demyelinating Disease:
• Multiple sclerosis
• Progressive multifocal leukoencephalopathy
Neurology: Vertigo Bootcamp.com

Peripheral Vertigo
Disease Pathophysiology Presentation Diagnostics Management

Benign paroxysmal Dislodged otoconia, <1 minute episode Clinical Epley (Canalith repositioning)
positional vertigo semicircular canals Worsened with head Dix-Hallpike maneuver maneuver
movements

Vestibular Neuritis Inflammatory reaction to Continuous worsened Clinical Supportive


vestibular nerve with head movements
Recent URI
(+ hearing loss if
labyrinthitis)

Meniere Disease Endolymph accumulation Minutes to hours Clinical, audiometry Acute: Vestibular suppressants
Episodic
Tinnitus, unilateral
hearing loss

Acoustic Neuroma Benign Schwann cell tumor Progressive worsening MRI with contrast Supportive vs surgical/radiation
at cerebellopontine angle Tinnitus, unilateral therapy
hearing loss
+/- CN V3, VII
involvement
≣ ⟽ ⟾
Item 1 of 1 Test Your Knowledge
◽" Mark Difficulty: ✪✪ Bootcamp.com
Question ID: 0017 Previous Next

A 57-year-old female presents to her primary care physician stating that


she had three episodes of dizziness over the past week. She states that
the first episode occurred when watering flowers in her garden and took a
few seconds to resolve. She thought she may have been dehydrated. She
reports two more episodes later in the week, at which time she awoke
from a partial sleep. She states that she rolled over to her left side in the
middle of the night and the dizziness began almost immediately. She
states that she is afraid to lay in her bed and has been consuming more
water than usual despite feeling nauseous after each episode. She denies
any tinnitus or hearing impairment. She also denies any recent illness and
states that she receives her flu-shot every year. Physical examination
findings are relatively unrevealing. Findings after otoscopy are shown. A
Dix-Hallpike maneuver is performed and a horizontal nystagmus with
reproducible “dizziness” is observed.

Which of the following is most likely the underlying pathophysiology


⚪ A. Post-infectious neuronal inflammatory reaction explaining this patient’s symptoms?
⚪ B. Dislodged endolymphatic debris
⚪ C. Accumulation of endolymphatic fluid
⚪ D. Obstruction of the eustachian tube
⚪ E. Emboli into the anterior inferior cerebellar artery

https://commons.wikimedia.org/wiki/File:TM_RIGHT_NORMAL.jpg
≣ ⟽ ⟾
Item 1 of 1 Test Your Knowledge
◽" Mark Difficulty: ✪✪ Bootcamp.com
Question ID: 0017 Previous Next

A 57-year-old female presents to her primary care physician stating that


she had three episodes of dizziness over the past week. She states that
the first episode occurred when watering flowers in her garden and took a
few seconds to resolve. She thought she may have been dehydrated. She
reports two more episodes later in the week, at which time she awoke
from a partial sleep. She states that she rolled over to her left side in the
middle of the night and the dizziness began almost immediately. She
states that she is afraid to lay in her bed and has been consuming more
water than usual despite feeling nauseous after each episode. She denies
any tinnitus or hearing impairment. She also denies any recent illness and
states that she receives her flu-shot every year. Physical examination
findings are relatively unrevealing. Findings after otoscopy are shown. A
Dix-Hallpike maneuver is performed and a horizontal nystagmus with
reproducible “dizziness” is observed.

Which of the following is most likely the underlying pathophysiology


⚪ A. Post-infectious neuronal inflammatory reaction explaining this patient’s symptoms?
$ B. Dislodged endolymphatic debris
⚪ C. Accumulation of endolymphatic fluid
⚪ D. Obstruction of the eustachian tube
⚪ E. Emboli into the anterior inferior cerebellar artery

https://commons.wikimedia.org/wiki/File:TM_RIGHT_NORMAL.jpg
OUTLINE

1. Pediatric Brain Tumors

Neurology: ●

A. Medulloblastoma
B. Pilocytic Astrocytoma
● C. Craniopharyngioma

Pediatric Brain ●
2.
D. Ependymoma
Infant Cranial Soft Tissue Injuries

Tumors ●


A. Cephalohematoma
B. Caput Succedaneum
C. Subgaleal Hemorrhage
Neurology: Pediatric Brain Tumors Bootcamp.com

• General Rules:
• Almost all infratentorial (exception: craniopharyngioma)
• Headaches worse at night or early morning
• +/- Noncommunicating hydrocephalus if in proximity to cerebellum and/or 4th ventricle

https://commons.wikimedia.org/wiki/File:Illu_tentorium.jpg
Neurology: Pediatric Brain Tumors Bootcamp.com

• Origin: Primitive neuroectoderm


• (“Primitive neuroectodermal tumors (PNETs))
• Severity: Aggressive
• Location: Cerebellum (vermis), infratentorial (usually)
• Microscopic: Primitive cells, poorly differentiated
• ↑ mitotic figures
• Small round blue anaplastic cells with minimal cytoplasm
• Homer-Wright pseudorosettes
• Presentation: Gait ataxia, noncommunicating hydrocephalus
• Association: Drop metastases

https://commons.wikimedia.org/wiki/File:1317_CFS_Circulation.jpg

https://commons.wikimedia.org/wiki/File:Micrograph_of_Homer_Wright_pseudorosettes.jpg

https://commons.wikimedia.org/wiki/File:Structure_of_a_Homer_Wright_pseudorosette.jpg

https://commons.wikimedia.org/wiki/File:Structure_of_a_rosette_in_pathology.jpg
Neurology: Pediatric Brain Tumors Bootcamp.com

• Origin: Astrocytes (glial cells)


• Neuroectoderm
• Glial Fibrillary Acidic Protein (GFAP) Marker
• Severity: Benign
• Location: Cerebellum, infratentorial (usually)
• Macroscopic: Cystic
• Microscopic: Well-differentiated, hair-like projections, Rosenthal fibers
• Presentation: Gait ataxia, emesis, failure to thrive

https://commons.wikimedia.org/wiki/File:Rosenthal_HE_40x.jpg
Neurology: Pediatric Brain Tumors Bootcamp.com

• Origin: Rathke’s Pouch


• Surface ectoderm
• Severity: Benign
• Location: Suprasellar region
• In proximity to pituitary
• Macroscopic: Cystic with dystrophic calcification (cholesterol crystals)
• Microscopic: “Wet keratin”
• Presentation: Bitemporal hemianopsia, hypopituitarism, central diabetes insipidus

https://commons.wikimedia.org/wiki/File:Illu_tentorium.jpg

https://radiopaedia.org/articles/craniopharyngioma

https://upload.wikimedia.org/wikipedia/commons/9/9e/Adamantinomatous_craniopharyngioma_-_intermed_mag.jpg
Neurology: Pediatric Brain Tumors Bootcamp.com

• Origin: Ependymal cells


• Neuroectoderm
• Severity: Variable
• Location: Peri-ventricular (4th ventricle), infratentorial (usually)
• Microscopic: Perivascular pseudorosettes
• Presentation: Intraventricular à obstructive hydrocephalus

https://commons.wikimedia.org/wiki/File:Ependymom_sag_FLAIR.jpg
https://commons.wikimedia.org/wiki/File:1317_CFS_Circulation.jpg
https://commons.wikimedia.org/wiki/File:Micrograph_of_perivascular_pseudorosettes.jpg
https://commons.wikimedia.org/wiki/File:Structure_of_a_perivascular_pseud
orosette.jpg
Neurology: Pediatric Brain Tumors Bootcamp.com

• Caput Succedaneum
• Between skin and epicranial aponeurosis
• Swelling crosses suture lines
• Generally benign
• Subgaleal Hemorrhage
• Between epicranial aponeurosis and periosteum
• Shearing of emissary veins
• Significant risk of hemorrhage
• Can cross suture lines
• Cephalohematoma
• Between periosteum and skull
• Does not cross suture lines
https://commons.wikimedia.org/wiki/File:Scalp_hematomas.jpg
≣ ⟽ ⟾
Item 1 of 1 Test Your Knowledge
◽" Mark Difficulty: ✪✪ Bootcamp.com
Question ID: 0018 Previous Next

A 12-year-old female is brought to the emergency department with family


following a visit earlier in the day with her pediatrician. She reportedly had
two episodes of non-bloody emesis during her check-up. Her parents
state that she has “migraine episodes” during the evening or during
bedtime. The patient has multiple bruises along her knees and elbows.
When questioned, the parents state that she has been in two falls when
riding her bicycle over the past month and has been “bumping into things”
more than usual. Imaging is performed in the emergency department. A
histopathologic specimen is subsequently obtained of the tumor identified
and shown below.

Which of the following features is most consistent with the origin of tumor
cells identified in the specimen?

⚪ A. Phagocytic cells of mesodermal origin


⚪ B. Benign tumor cells of surface ectodermal origin
⚪ C. Glial fibrillary acidic protein positivity
⚪ D. Simple columnar glial cells known to produce cerebrospinal fluid
⚪ E. S100 positive cells of neural crest origin

https://commons.wikimedia.org/wiki/File:Ependymoma_pseudorosette.jpg
https://neuropathology-web.org/chapter7/chapter7dEpendymoma.html
≣ ⟽ ⟾
Item 1 of 1 Test Your Knowledge
◽" Mark Difficulty: ✪✪ Bootcamp.com
Question ID: 0018 Previous Next

A 12-year-old female is brought to the emergency department with family


following a visit earlier in the day with her pediatrician. She reportedly had
two episodes of non-bloody emesis during her check-up. Her parents
state that she has “migraine episodes” during the evening or during
bedtime. The patient has multiple bruises along her knees and elbows.
When questioned, the parents state that she has been in two falls when
riding her bicycle over the past month and has been “bumping into things”
more than usual. Imaging is performed in the emergency department. A
histopathologic specimen is subsequently obtained of the tumor identified
and shown below.

Which of the following features is most consistent with the origin of tumor
cells identified in the specimen?

⚪ A. Phagocytic cells of mesodermal origin


⚪ B. Benign tumor cells of surface ectodermal origin
⚪ C. Benign tumor cells of neural crest cell origin
$ D. Simple columnar glial cells known to produce cerebrospinal fluid
⚪ E. S100 positive cells of neural crest origin

https://commons.wikimedia.org/wiki/File:Ependymoma_pseudorosette.jpg
https://neuropathology-web.org/chapter7/chapter7dEpendymoma.html
OUTLINE

1. Adult Primary Brain Tumors

Neurology: ●

A. Glioblastoma Multiforme
B. Vestibular Schwannoma
● C. Oligodendroma

Adult Primary ●

D. Meningioma
E. Primary Central Nervous System Lymphoma

Brain Tumors
Neurology: Adult Primary Brain Tumors Bootcamp.com

• General Rules:
• Almost all supratentorial (exception: vestibular schwannoma, hemangioblastoma)
• Headaches worse at night or early morning
• GFAP +: Marker of glial origin, classic case = GBM
• Technically, observed in most forms of adult primary brain tumors
• NOT observed in meningiomas
• Synaptophysin +: Marker of neuroendocrine cells
• NOT observed in meningiomas

https://commons.wikimedia.org/wiki/File:Illu_tentorium.jpg
Neurology: Adult Primary Brain Tumors Bootcamp.com

• Origin: Astrocytes (glial cells or pluripotent neural stem cells)


• Neuroectoderm
• Severity: Aggressive
• Location: Cerebral hemispheres
• Butterfly glioma: crossing of corpus callosum
• Microscopic: Primitive cells, poorly differentiated
• Pseudopalisading necrosis
• Peripheral vascular proliferation (neovascularization)
• ↑ mitotic figures
• Molecular: Overexpression epidermal growth factor receptor
(EGFR)
• Presentation: New onset seizures, headache, FNDs

https://commons.wikimedia.org/wiki/File:GBM_pseudopalisading_necrosis.jpg
https://radiopaedia.org/articles/butterfly-glioma?lang=us
Neurology: Adult Primary Brain Tumors Bootcamp.com

• Origin: Schwann cells (PNS)


• Neural crest cell
• Severity: Benign
• Location: Cerebellopontine angle (infratentorial)
• Unilateral: Spontaneous
• Bilateral: Neurofibromatosis type 2
• Molecular: S100 positive (neural crest cell origin)
• Presentation:
• Vestibular portion of CN VIII: Disequilibrium, vertigo
• Cochlear portion of CN VIII: Sensorineural hearing loss, tinnitus
• Facial nerve (CN VII): Facial muscle motor impairment, impaired corneal reflex
• Trigeminal nerve (CN V): Facial muscle numbness

https://commons.wikimedia.org/wiki/File:Akustikusneurinom_Mrt.jpg
Neurology: Adult Primary Brain Tumors Bootcamp.com

• Origin: Oligodendrocytes (CNS)


• Severity: Benign
• Location: Cerebral hemisphere
• Frontal lobe is classic
• Microscopic: “Fried egg” appearance to cells
• Capillaries à “Chicken-wire” pattern
• Presentation: Seizures, headaches, FNDs, (if frontal lobe à personality changes)

https://www.neurocirugiabarcelona.com/en/pathologies/brain-tumours/glial-
tumours/oligodendrogliomas/

https://commons.wikimedia.org/wiki/File:Flickr_-_cyclonebill_-_Vagtel-spejl%C3%A6g.jpg
https://commons.wikimedia.org/wiki/File:Oligodendroglioma1_high_mag.jpg
Neurology: Adult Primary Brain Tumors Bootcamp.com

• Origin: Arachnoid cells


• Severity: Benign
• Slow growing
• Location: Parasagittal region of cerebral hemispheres

In proximity to dural reflections (falx cerebri, foramen magnum)
• Macroscopic: Extra-axial
• Microscopic: Psammoma bodies
• Eosinophilic whorls with layered appearance (onion)
• Molecular: Negative for GFAP and synaptophysin
• Presentation: Asymptomatic vs seizures, headaches, FNDs

https://commons.wikimedia.org/wiki/File:MRIMeningioma.png
https://commons.wikimedia.org/wiki/Category:Psammoma_bodies#/media/File:Psammoma_bodies.jpg

https://commons.wikimedia.org/wiki/File:Meningioma_high_mag.jpg
Neurology: Adult Primary Brain Tumors Bootcamp.com

• Origin: B-cells (typically)


• Common subtype: Diffuse large B-cell lymphoma
• Severity: Aggressive
• Macroscopic: Lesions throughout white and gray matter
• Ring-enhancing lesion on CT
• Microscopic: Aggregates of atypical lymphoid cells
• Molecular: CD20, CD79a
• Presentation: Immunocompromised (poorly controlled HIV/AIDs), FNDs, seizure
• Association: Epstein—Barr virus

https://radiopaedia.org/cases/primary-cns-lymphoma-2
≣ ⟽ ⟾
Item 1 of 1 Test Your Knowledge
◽" Mark Difficulty: ✪✪✪✪ Bootcamp.com
Question ID: 0019 Previous Next

A 44-year-old female presents to the emergency room after experiencing


an episode of loss of consciousness with generalized uncontrollable and
rhythmic muscle contractions. Her husband reports that she has been
unable speak well since the incident. He believes that she may be
embarrassed due to the episode of bladder incontinence she suffered
when paramedics arrived. Lacerations on the tongue are noted and the
patient is visibly aphasic. A review of the patient’s medical history reveals
a history of hypothyroidism and no history of previous seizure episodes.
Her husband states that she has no family history of seizures, and she
does not consume alcohol. A head CT and subsequent biopsy of identified
mass lesion is performed and shown.

Which of the following is most consistent with the patient’s tumor?

⚪ A. Derived from tissue of neuroectoderm


⚪ B. Derived from tissue of neural crest cell origin
⚪ C. Glial fibrillary acidic protein positive
⚪ D. Synaptophysin positive
⚪ E. Strong staining for CD20

https://commons.wikimedia.org/wiki/File:Contrast_enhanced_meningioma.jpg
https://commons.wikimedia.org/wiki/File:Meningioma_showing_Psammoma_body.jpg
≣ ⟽ ⟾
Item 1 of 1 Test Your Knowledge
◽" Mark Difficulty: ✪✪✪✪ Bootcamp.com
Question ID: 0019 Previous Next

A 44-year-old female presents to the emergency room after experiencing


an episode of loss of consciousness with generalized uncontrollable and
rhythmic muscle contractions. Her husband reports that she has been
unable speak well since the incident. He believes that she may be
embarrassed due to the episode of bladder incontinence she suffered
when paramedics arrived. Lacerations on the tongue are noted and the
patient is visibly aphasic. A review of the patient’s medical history reveals
a history of hypothyroidism and no history of previous seizure episodes.
Her husband states that she has no family history of seizures, and she
does not consume alcohol. A head CT and subsequent biopsy is
performed and shown.

Which of the following is most consistent with the patient’s mass lesion?

⚪ A. Derived from tissue of neuroectoderm


$ B. Derived from tissue of neural crest cell origin
⚪ C. Glial fibrillary acidic protein positive
⚪ D. Synaptophysin positive
⚪ E. Strong staining for CD20

https://commons.wikimedia.org/wiki/File:Contrast_enhanced_meningioma.jpg
https://commons.wikimedia.org/wiki/File:Meningioma_showing_Psammoma_body.jpg
OUTLINE

1. Types of Ischemic Cerebrovascular Accidents

Neurology: ●

A. Thrombotic
B. Embolic
● C. Global

Ischemic 2.

Histopathology
A. Liquefactive Necrosis

Cerebrovascular
B. Histologic Changes by Time
3. Cerebrovascular Accident by Cerebral Territory
● A. Middle Cerebral Artery

Accidents
● B. Anterior Cerebral Artery
● C. Posterior Cerebral Artery
● D. Lenticulostriate Arteries
4. Lacunar Infarction
● A. General Overview
● B. Pure Motor, Pure Sensory, Sensorimotor Variants
● C. Subthalamic Infarction
5. Brainstem Infarction
● A. Medial Medullary Syndrome
● B. Lateral Medullary Syndrome
● C. Lateral Pontine Syndrome
● D. ”Locked In” Syndrome
6. Basics of Cerebrovascular Accident Management
● A. Acute Management- tPA
● B. Secondary Prevention
Neurology: Cerebrovascular Accidents Bootcamp.com

CVA

Ischemic Hemorrhagic

Thrombotic Subarachnoid Other Intracranial Intraparenchymal Intraventricular

Epidural
Embolic Lobar
Hematoma

Subdural
Lacunar Lacunar
hematoma

Hypoperfusion
Neurology: Cerebrovascular Accidents Bootcamp.com

Transient Ischemic Attack


FNDs without tissue infarction
Normal imaging
Typically resolve without intervention <24 hours

• Thrombotic: Remember UMN Signs!


• Etiology à Atherosclerotic plaques à vessel occlusion Hyperreflexia
Hypertonia
• Embolic: Spasticity (Clasp-knife)
Babinski sign (upgoing plantar reflex)
• Etiology à Atrial fibrillation, bacterial endocarditis
• Classic areas affected à MCA, ACA, or PCA division (large vessel disease)
• Asymmetric pattern of scattered infarcts
• Hypoperfusion (Global):
• Watershed areas affected
• Wedge-appearing areas of necrosis bilaterally (autopsy)
• Hippocampus = vulnerable to hypoxia

https://commons.wikimedia.org/wiki/File:Human_brain_frontal_(coronal)_section.JPG
Neurology: Cerebrovascular Accidents Bootcamp.com

https://commons.wikimedia.org/wiki/File:MCA-Stroke-Brain-Human-2.JPG

• Liquefactive Necrosis:
• Irreversible injury approximately 4-5 minutes of hypoxia
• Neural cell irreversible injury à hydrolytic enzymes (lysosomes) à liquefactive necrosis (weeks)
• Microglia digestion à cystic spaces surrounded by gliosis (months), pale infarct

Cerebrovascular Accident Histopathology by Timeline


Time Post-Injury Pathophysiology Histologic Presentation
Failure of ATP-gated ion channels Minimal changes
1 hour ↑ glutamate Cytotoxic edema

Activity and dissolution of Nissl bodies Red neurons à eosinophilic cytoplasm (loss of Nissl bodies)
12-24 hours Irreversible chromatin condensation Pyknotic nuclei à small, basophilic nuclei

Neutrophilic invasion Liquefactive necrosis


Disruption of tight junctions of BBB ↑ local neutrophil density
1-3 days Vasogenic edema

Macrophage and microglial invasion ↑ local macrophage and microglial density


3-7 days Phagocytosis ensues

Reactive gliosis Astrocyte proliferation, glial hypertrophy


1-2 weeks Vascular proliferation at peripheral sites to necrosis

Dense astrocytic processes surrounding cavity of necrotic Cystic spaces surrounded by glial scarring
2 weeks and beyond debris
Neurology: Cerebrovascular Accidents Bootcamp.com

Cerebrovascular Accident Presentation by Territory


Artery High Yield Territories Classic Lesion Presentation

Basal ganglia Chorea, tremor, bradykinesia, rigidity, altered behavior


Lenticulostriate Internal capsule (posterior limb) C/L motor and sensory impairment

Primary motor cortex- Frontal C/L upper extremity and lower face, motor and sensory impairment
Broca’s area (D)-Frontal Nonfluent aphasia (Expressive)
Wernicke’s area (D)-Temporal Fluent aphasia (Receptive)
MCA Angular gyrus (D)- Parietal Gerstmann Syndrome
(ND)- Parietal C/L: Hemineglect
Frontal eye fields- Frontal Conjugate deviation to side of lesion
Optic radiations C/L homonymous hemianopia or quadrantanopia

Anteromedial primary motor and sensory cortex C/L lower extremity, motor and sensory impairment
ACA B/L urinary incontinence, altered behavior (abulia)

Primary visual cortex- Occipital C/L homonymous hemianopia with macular sparing
PCA

(D): Dominant (usually left)


(ND): Non-dominant (usually right)
C/L: Contralateral
B/L: Bilateral
Neurology: Cerebrovascular Accidents Bootcamp.com

https://commons.wikimedia.org/wiki/File:CT_of_lacunar_strokes.jpg
https://commons.wikimedia.org/wiki/File:Blausen_0076_BasalGanglia.png

• Artery: Lenticulostriate arteries


• Association: #1 Chronic poorly controlled hypertension
• Location: Basal ganglia, posterior limb internal capsule,
cerebellum
• Presentation:
• Internal capsule: Contralateral hemiparesis +/- hemisensory loss
• Basal ganglia: Tremor, bradykinesia, chorea, rigidity, altered behavior
• Unusual to observe cortical signs
• Lacunar Ischemia: Hypertensive arteriolar sclerosis
• Microatheroma à lipid-laden macrophage accumulation, commonly at
branch points
• Lipohyalinosis à foamy macrophages, vascular wall thickening
• Lacunar Hemorrhage: Hypertensive vasculopathy
• Charcot-Bouchard aneurysms
Neurology: Cerebrovascular Accidents Bootcamp.com

• Pure Motor: Contralateral upper and lower extremity, face


• Posterior limb of internal capsule (most common)
• Pure Sensory: Contralateral upper and lower extremity, face
• Ventroposterolateral and/or ventroposteriomedial thalamus (common)
• Posterior limb of internal capsule (rare)
• Mixed Sensorimotor: Contralateral upper and lower extremity, face
• Posterior limb of internal capsule (common)
• Hemiballismus: Contralateral involuntary large flinging movements of extremities
• Subthalamic nucleus
Neurology: Cerebrovascular Accidents Bootcamp.com

(D): Dominant (usually left)


(ND): Non-dominant (usually right)
• Medial Medullary (Dejerine) Syndrome: I/L: Ipsilateral
• Anterior spinal artery or vertebral artery
C/L: Contralateral
• CN XII à I/L tongue palsy (deviation to side of lesion) B/L: Bilateral
• Medial tracts à C/L weakness of limbs, C/L loss of vibration and proprioception
• Lateral Medullary (Wallenberg) Syndrome:
• Posterior inferior cerebellar artery (PICA) or vertebral artery
• CN VIII à vertigo, nystagmus, hearing loss
• CN IX, X à dysphagia, dysphonia, impaired gag reflex
• Lateral tracts à loss of temperature and pain sensation I/L face, C/L trunk and limbs, I/L ataxia, dysmetria,
dysdiadochokinesia
• Sympathetic (lateral) tract à Horner syndrome I/L (miosis, ptosis, anhidrosis)
• Lateral Pontine Syndrome:
• Anterior inferior cerebellar artery (AICA)
• CN VII à facial muscle weakness, hyperacusis, loss of taste anterior 2/3 of tongue
• CN VIII à vertigo, nystagmus, hearing loss
• Lateral tracts à loss of temperature and pain sensation I/L face, C/L trunk and limbs, I/L ataxia, dysmetria,
dysdiadochokinesia
• Sympathetic (lateral) tract à Horner syndrome I/L (miosis, ptosis, anhidrosis)
Neurology: Cerebrovascular Accidents Bootcamp.com

• Artery: Basilar artery (bilateral ventral pontine CVA)


• Location: Ventral pons
• Presentation:
• Quadriparesis
• Horizontal gaze palsy (vertical eye movements and voluntary blinking preserved)
• Anarthria, dysphagia
• Cheyne-Stokes's respirations
• Osmotic Demyelination Syndrome (Central Pontine Myelinolysis):
• Etiology: rapid overcorrection of chronic hyponatremia
• Symmetric pontine demyelination on MRI
Neurology: Cerebrovascular Accidents Bootcamp.com

• Anti-platelet therapy: Aspirin, clopidogrel


• HMG-CoA Reductase Inhibitor: Atorvastatin, rosuvastatin, pravastatin
• Lifestyle Modifications: Diet, exercise, smoking cessation

• Tissue Plasminogen Activator (tPA)


• < 4.5 hours of onset of symptoms
≣ ⟽ ⟾
Item 1 of 1 Test Your Knowledge
◽" Mark Difficulty: ✪✪ Bootcamp.com
Question ID: 0020 Previous Next

A 59-year-old male presents to the emergency room stating, “I can’t feel my


left arm or leg.” He reports that he suddenly felt weak while washing dishes
after dinner and nearly lost his balance. He denies falling and states that he
was able to sit down immediately after symptom onset. He denies any
confusion or loss of consciousness. He states that he has no difficulty
speaking and denies any dysphagia. He has a long history of chronic
hypertension and no previous medical or family history of cerebrovascular
disease or hemorrhage. He reports that he has not refilled his
antihypertensive prescription medications because of a demanding work
schedule over the past month. Physical examination reveals generalized
sensory loss on the left upper and lower extremities and 4/5 muscle strength
of the left thigh, shoulder, and biceps. No abnormal involuntary motor
movements are noted. Extraocular eye movements are intact bilaterally, and
no nystagmus is observed. Non-contrast CT of the head is performed and
shown.

Which of the following findings would be most consistent with this patient’s
presentation at this time?
⚪ A. Infarction and cytotoxic edema within the subthalamic nucleus
⚪ B. Lipohyalinosis of vessels supplying the posterior limb of the internal capsule
⚪ C. Symmetric pontine demyelination represented bilaterally
⚪ D. Cavitary lesion overlying the lenticulostriate artery distribution surrounded by glial hypertrophy
⚪ E. Bilateral renal enlargement with diffuse cystic disease
https://upload.wikimedia.org/wikipedia/en/0/04/Brain_CT_scan.jpg
≣ ⟽ ⟾
Item 1 of 1 Test Your Knowledge
◽" Mark Difficulty: ✪✪ Bootcamp.com
Question ID: 0020 Previous Next

A 59-year-old male presents to the emergency room stating, “I can’t feel my


left arm or leg.” He reports that he suddenly felt weak while washing dishes
after dinner and nearly lost his balance. He denies falling and states that he
was able to sit down immediately after symptom onset. He denies any
confusion or loss of consciousness. He states that he has no difficulty
speaking and denies any dysphagia. He has a long history of chronic
hypertension and no previous medical or family history of cerebrovascular
disease or hemorrhage. He reports that he has not refilled his
antihypertensive prescription medications because of a demanding work
schedule over the past month. Physical examination reveals generalized
sensory loss on the left upper and lower extremities and 4/5 muscle strength
of the left thigh, shoulder, and biceps. No abnormal involuntary motor
movements are noted. Extraocular eye movements are intact bilaterally, and
no nystagmus is observed. Non-contrast CT of the head is performed and
shown.

Which of the following findings would be most consistent with this patient’s
presentation at this time?
⚪ A. Infarction and cytotoxic edema within the subthalamic nucleus
$ B. Lipohyalinosis of vessels supplying the posterior limb of the internal capsule
⚪ C. Symmetric pontine demyelination represented bilaterally
⚪ D. Cavitary lesion overlying the lenticulostriate artery distribution surrounded by glial hypertrophy
⚪ E. Bilateral renal enlargement with diffuse cystic disease
https://upload.wikimedia.org/wikipedia/en/0/04/Brain_CT_scan.jpg
OUTLINE

1. Cerebral Blood Supply

Neurology: ●
2.
A. Circle of Willis
Cerebrovascular Aneurysm
● A. Saccular Aneurysm
Aneurysms and ●

B. Charcot-Bouchard Aneurysm
C. Posterior Communicating Artery Aneurysm

Hemorrhagic ●
3.

D. Anterior Communicating Artery Aneurysm
Intracranial Hemorrhage
A. Epidural Hematoma

Cerebrovascular ●

B. Subdural Hematoma
C. Subarachnoid Hemorrhage
● D. Intraparenchymal Hemorrhage

Accidents ● E. Intraventricular Hemorrhage


Neurology: Cerebrovascular Accidents Bootcamp.com

CVA

Ischemic Hemorrhagic

Thrombotic Subarachnoid Other Intracranial Intraparenchymal Intraventricular

Epidural
Embolic Lobar
Hematoma

Subdural
Lacunar Lacunar
hematoma

Hypoperfusion
Neurology: Brain Blood Supply and Aneurysms Bootcamp.com

https://commons.wikimedia.org/wiki/File:2123_Arteries_of_the_Brain.jpg
https://case.edu/med/neurology/NR/MagnResnAngiogrMRA/MagnResnAngiogrMRA.htm
Neurology: Brain Blood Supply and Aneurysms Bootcamp.com

https://commons.wikimedia.org/wiki/File:2123_Arteries_of_the_Brain.jpg

• Saccular:
• Arise at branch points
• Anterior communicating artery + anterior cerebral artery junction = MC
• Rupture à Subarachnoid hemorrhage à FNDs not usually present
• Charcot-Bouchard: *if present - would depend on aneurysm size/
location*
• Association with chronic hypertension and diabetes
• Lenticulostriate vessels affected
• Rupture à Lacunar Hemorrhage à FNDs (may vary)
Neurology: Brain Blood Supply and Aneurysms Bootcamp.com

https://commons.wikimedia.org/wiki/File:2123_Arteries_of_the_Brain.jpg

• Posterior Communicating Artery Aneurysm:


• Oculomotor nerve (CN III) palsy à dilated (first) à “down and out” pupil, ptosis
• Anterior Communicating Artery Aneurysm:
• Optic chiasm à Bitemporal hemianopia
Neurology: Brain Blood Supply and Aneurysms Bootcamp.com
https://commons.wikimedia.org/wiki/File:EpiduralHematoma.jpg
https://commons.wikimedia.org/wiki/File:Subduralandherniation.PNG

https://commons.wikimedia.org/wiki/File:SubarachnoidP.png

• Epidural Hematoma: Remember signs of ↑ ICP!


• Middle meningeal artery (branch of maxillary artery) Papilledema
• Area between skull and dura mater Cushing Reflex
• Transient LOC à Lucid interval à LOC
Headache
• Biconvex hematoma (lens-shape)
• Uncal herniation à ipsilateral dilated pupil, ipsilateral lateral rectus palsy Lateral rectus palsy, I/L mydriasis
• Subdural Hematoma:
• Bridging cortical veins
• Area between dura and arachnoid mater
• Acute vs chronic, elderly patient, shaken baby syndrome
• Crescent hematoma
• Subarachnoid Hemorrhage:
• Rupture of saccular (berry) aneurysm or AV malformation
• Area between arachnoid and pia mater
• ”Worst headache of life”, thunderclap headache, neck stiffness/pain
• Mental status progressively worsens
• Blood pooling in basal cisterns
• Focal neurologic deficits are unusual (vs intracerebral hemorrhage)

Head Imaging Post-Trauma à CT scan without contrast


Neurology: Brain Blood Supply and Aneurysms Bootcamp.com

Suspicion for
SAH
• Cerebral Vasospasm
• Cerebral ischemia delayed from initial SAH
• 4-12 days after initial SAH Head CT
• Focal neurologic deficits (unlike with initial SAH) without contrast
• Head CT without contrast usually negative
• Prevent with Nimodipine Blood in basal
• Rebleeding generally first 24 hours cisterns = Negative
Diagnostic
• Associations
• ADPKD, Ehler’s Danlos
Lumbar
Puncture

Xanthochromia
Negative (or ↑ RBC)

Unlikely SAH Likely SAH


Neurology: Brain Blood Supply and Aneurysms Bootcamp.com

Remember signs of ↑ ICP!


Papilledema
Cushing Reflex
• Lobar Hemorrhage: Headache
Lateral rectus palsy, I/L mydriasis
• Signs of ↑ ICP, FNDs
• Elderly patient with cerebral amyloid angiopathy
• Temporal lobe à Uncal (transtentorial) herniation
• High tendency for recurrence
• Lacunar Hemorrhage:
• Charcot-Bouchard aneurysms
• Hypertensive vasculopathy
• Putamen = MC site, thalamus = 2nd MC site
• Internal capsule: Contralateral hemiparesis +/- hemisensory loss
• Basal ganglia: Tremor, bradykinesia, chorea, rigidity, altered behavior
Neurology: Brain Blood Supply and Aneurysms Bootcamp.com

• Association: Premature (or low birth weight) newborn


• Location: Germinal matrix
• Presentation:
• First few days after birth
• Bulging anterior fontanelle
• New onset FNDs +/- seizures and other signs of ↑ ICP
• Differential:
• Forceps or vacuum assisted delivery à
≣ ⟽ ⟾
Item 1 of 1 Test Your Knowledge
◽" Mark Difficulty: ✪✪ Bootcamp.com
Question ID: 0021 Previous Next

https://case.edu/med/neurology/NR/MagnResnAngiogrMRA/MagnResnAngiogrMRA.htm

A 33-year-old female presents to the emergency room with gross


D hematuria. She states that she has been experiencing progressively
E worsening flank pain bilaterally over the past six months. She denies any
C recent trauma but reports that she nearly collided with another vehicle
while driving a few weeks ago because the other car “came out of
nowhere”. Since that time, she has avoided driving and playing sports
with friends. Her family history is significant for cerebral hemorrhage in
her father and grandmother. Blood pressure is 164/80, heart rate 78/min,
A B respiratory rate 12/min. Peripheral visual field findings are shown. Renal
ultrasound reveals significant diffuse cystic disease in the bilateral
kidneys.

Which of the following single lesion sites would best explain this patient’s
presentation?

⚪ A.
⚪ B.
⚪ C.
⚪ D.
⚪ E.
≣ ⟽ ⟾
Item 1 of 1 Test Your Knowledge
◽" Mark Difficulty: ✪✪ Bootcamp.com
Question ID: 0021 Previous Next

https://case.edu/med/neurology/NR/MagnResnAngiogrMRA/MagnResnAngiogrMRA.htm

A 33-year-old female presents to the emergency room with gross


D hematuria. She states that she has been experiencing progressively
E worsening flank pain bilaterally over the past six months. She denies any
C recent trauma but reports that she nearly collided with another vehicle
while driving a few weeks ago because the other car “came out of
nowhere”. Since that time, she has avoided driving and playing sports
with friends. Her family history is significant for cerebral hemorrhage in
her father and grandmother. Blood pressure is 164/80, heart rate 78/min,
A B respiratory rate 12/min. Peripheral visual field findings are shown. Renal
ultrasound reveals significant diffuse cystic disease in the bilateral
kidneys.

Which of the following single lesion sites would best explain this patient’s
presentation?

⚪ A.
⚪ B.
⚪ C.
⚪ D.
$ E.
OUTLINE

1. Anatomical Considerations

Neurology: ●

A. Sagittal Brain MRI
B. Ventricular Brain Anatomy
● C. Flow of Cerebrospinal Fluid

Cerebellum and ●

D. Cerebellar Anatomical Zones
E. Area Postrema

Ventricles 2.


Hydrocephalus
A. Noncommunicating Hydrocephalus
B. Communicating Hydrocephalus
● C. Normal Pressure Hydrocephalus
● D. Hydrocephalus Ex Vacuo
● E. Pseudotumor Cerebri
3. Sensory vs Cerebellar Ataxia
● A. Sensory Ataxia
● B. Cerebellar Ataxia
● C. Romberg Test
4. Cerebellar Pathology
● A. Ataxia Telangiectasia
● B. Friedreich Ataxia
● C. Paraneoplastic Cerebellar Degeneration
● D. Dandy-Walker Malformation
● E. Arnold-Chiari Malformation
Neurology: Cerebellum and Ventricles Bootcamp.com

https://radiopaedia.org/articles/sagittal-midline-of-the-brain-an-approach-1

• Sagittal MRI Key Landmarks:


• Thalamus
• Pons
• Cerebellum
• Area Postrema
• Corpus Callosum
• Lateral Ventricle
• 4th Ventricle
Neurology: Cerebellum and Ventricles Bootcamp.com

Exits via foramina


CSF produced at Interventricular Cerebral aqueduct of Luschka or Subarachnoid Arachnoid
choroid plexus Lateral ventricle foramina of Monro Third Ventricle (of Sylvius) Fourth ventricle foramen of space granulations Venous circulation
Magendie

https://commons.wikimedia.org/wiki/File:CSF_circulation.png
Neurology: Cerebellum and Ventricles Bootcamp.com
https://commons.wikimedia.org/wiki/File:Papilledema.jpg
https://radiopaedia.org/articles/normal-pressure-hydrocephalus
https://radiopaedia.org/images/15747811?case_id=39554
https://commons.wikimedia.org/wiki/File:Blausen_0896_Ventricles_Brain.png

• Noncommunicating (Obstructive) Hydrocephalus:


• ↑ ICP, ventriculomegaly proximal to obstruction
• Etiology: Scarring, mass lesions
• Malformations: Arnold-Chiari malformation, Dandy-Walker malformation
• Infectious: CMV, meningitis, toxoplasmosis
• Other classic causes: Intraventricular bleed
• Communicating Hydrocephalus:
• ↑ ICP, diffuse ventriculomegaly
• Etiology: Scarring due to ↓ resorption at arachnoid granulations, ↑ production
• Classic Causes: Subarachnoid hemorrhage, choroid plexus papilloma
• Normal Pressure Hydrocephalus:
• Normal ICP, ventriculomegaly out of proportion to cortical atrophy
• Etiology: Impaired CSF absorption
• Triad: ataxia (“magnetic gait”), urinary incontinence, dementia
• Hydrocephalus Ex Vacuo:
• Normal ICP, ventriculomegaly in proportion to cortical atrophy
• Etiology: Diffuse cortical atrophy
• Association: Dementia (AD), AIDS, Huntington disease
• Pseudotumor Cerebri (Idiopathic Intracranial Hypertension):
• ↑ ICP, normal ventricular size
• Association: Young, obese female, excessive vitamin A
Neurology: Cerebellum and Ventricles Bootcamp.com

• Midline Cerebellar Lesions: Truncal ataxia, wide-based gait


• Lateral Cerebellar Lesions: Ipsilateral dysmetria, intention tremor
• Inferior Cerebellar Lesions: Vertigo, nystagmus

Cerebellum Overview *May also


observe with
lesions to
Cerebellar Classic inferior
Location Primary Function vermis
Territory Lesion(s)
Motor via medial descending motor Truncal ataxia
Vermis Midline
systems Wide-based gait https://commons.wikimedia.org/wiki/File:CerebellumDiv.png
https://dizziness-and-balance.com/disorders/central/cerebellar/cerebellar.htm

Balance, eye movements via


Flocculonodular vestibular nuclei and medial
Inferiorly Vertigo*, nystagmus*
Lobe longitudinal fasciculus

Intermediate Motor via lateral descending motor


Lateral to vermis Intention tremor
Hemisphere systems
Ipsilateral
dysdiadochokinesia
Lateral to
Lateral intermediate
Ipsilateral dysmetria
Motor planning
Hemisphere hemisphere
Neurology: Cerebellum and Ventricles Bootcamp.com

• Cerebellar Ataxia:
• Classic Regions Involved: Cerebellum
• Etiology: Ataxia Telangiectasia, cerebellar tumor
• Sensory Ataxia:
• Classic Regions Involved: Peripheral nervous system, DCML
• Etiology: Vitamin B12 deficiency, tertiary syphilis (tabes dorsalis)
Romberg Test

Eyes open Eyes closed

Minimal changes to
Unsteadiness unsteadiness
Unsteadiness No unsteadiness worsening or failure No unsteadiness
present present (compared with eyes present
to maintain posture open)

Unlikely sensory
Unspecified form of Unlikely cerebellar Sensory and/or and/or vestibular
ataxia ataxia vestibular ataxia Cerebellar ataxia
ataxia
Neurology: Cerebellum and Ventricles Bootcamp.com

https://radiopaedia.org/cases/dandy-walker-malformation-11
https://radiopaedia.org/articles/chiari-ii-malformation
https://commons.wikimedia.org/wiki/File:Dermoscopy_nodular_basal_cell_carcinoma.jpg

• Ataxia Telangiectasia:
• Triad: Cerebellar ataxia, telangiectasias (spider angiomas), IgA deficiency
• Mutation à ATM gene
• Friedreich Ataxia:
• Adolescent or older child
• Deficiency of frataxin, trinucleotide repeat expansion of GAA on chromosome 9
• UMN and LMN signs
• Progressive cerebellar and sensory ataxia
• Association: Scoliosis, pes cavus, hammertoes, diabetes mellitus, HOCM
• Paraneoplastic Cerebellar Degeneration:
• History of cancer (Breast, ovarian, uterine, small cell lung)
• Rapid deterioration and worsening of cerebellar symptoms
• Anti-Yo, anti-Hu, anti-P/Q
• Dandy-Walker Malformation:
• Infant with developmental delay
• Hypoplasia (or complete absence) of cerebellar vermis
• Enlarged posterior fossa and dilation of fourth ventricle
• Arnold-Chiari Malformation (Chiari II):
• Infant with developmental delay à obstructive hydrocephalus
• Inferior displacement of cerebellar tonsils and vermis
• Association with myelomeningocele
Neurology: Cerebellum and Ventricles Bootcamp.com

• Area Postrema:
• Chemoreceptor trigger zone
• Direct stimulation via substance P à NK1 receptors in brainstem (↓ BBB)
• Indirect stimulation via serotonin à 5-HT3 receptors on vagal afferents
• Antiemetic Pharmacology
• Ondansetron à selective 5-HT3 receptor antagonist
• Aprepitant à NK1 receptor antagonist
≣ ⟽ ⟾
Item 1 of 1 Test Your Knowledge
◽" Mark Difficulty: ✪ Bootcamp.com
Question ID: 0022 Previous Next

A 77-year-old female with a past medical history of atrial fibrillation and


hypertension presents to the emergency department with an acutely
worsening occipital headache and nausea. She states that she had one
episode of non-bloody emesis en route to the hospital. She denies any
vertigo, tinnitus, or vision changes. She admits to a 34-pack year history
of tobacco use. Her medications include warfarin, lisinopril, and a
multivitamin. Her temperature is 97.6 F (36.4 C), blood pressure is
194/90, heart rate is 94/min, respirations are 12/min. Electrocardiogram
reveals sinus rhythm. Laboratory studies are as follows:

Hemoglobin 14.2 g/dL


Leukocytes 5,000/mm3
Platelets 285,000/mm3
Prothrombin time prolonged
Activated partial thromboplastin time normal
⚪ A. Coarse hand tremor worsened with targeted movement
⚪ B. Pill-rolling hand tremor worsened with stress
⚪ C. Left-sided upper extremity weakness Noncontrast CT of the head is shown. Which of the following findings
⚪ D. Bilateral pes cavus would most likely be observed given the patient’s presentation?
⚪ E. Urinary incontinence
⚪ F. Hyperorality

https://radiopaedia.org/articles/cerebellar-haemorrhage
≣ ⟽ ⟾
Item 1 of 1 Test Your Knowledge
◽" Mark Difficulty: ✪ Bootcamp.com
Question ID: 0022 Previous Next

A 77-year-old female with a past medical history of atrial fibrillation and


hypertension presents to the emergency department with an acutely
worsening occipital headache and nausea. She states that she had one
episode of non-bloody emesis en route to the hospital. She denies any
vertigo, tinnitus, or vision changes. She admits to a 34-pack year history
of tobacco use. Her medications include warfarin, lisinopril, and a
multivitamin. Her temperature is 97.6 F (36.4 C), blood pressure is
194/90, heart rate is 94/min, respirations are 12/min. Electrocardiogram
reveals sinus rhythm. Laboratory studies are as follows:

Hemoglobin 14.2 g/dL


Leukocytes 5,000/mm3
Platelets 285,000/mm3
Prothrombin time prolonged
Activated partial thromboplastin time normal
$ A. Coarse hand tremor worsened with targeted movement
⚪ B. Pill-rolling hand tremor worsened with stress
⚪ C. Left-sided upper extremity weakness Noncontrast CT of the head is shown. Which of the following findings
⚪ D. Bilateral pes cavus would most likely be observed given the patient’s presentation?
⚪ E. Urinary incontinence
⚪ F. Hyperorality

https://radiopaedia.org/articles/cerebellar-haemorrhage
OUTLINE

1. Hypothalamus

Neurology: ●

A. Hypothalamic Nuclei
B. Jet Lag Syndrome
● C. Kallman Syndrome

Diencephalon ●

D. Wernicke-Korsakoff Syndrome
E. Disorders of Temperature Regulation
2. Thalamus
● A. Thalamic Nuclei
● B. Thalamic Syndrome
● C. Lacunar Stroke
● D. Hydrocephalus Ex Vacuo
● E. Pseudotumor Cerebri
3. Pineal Gland
● A. Suprachiasmatic Nucleus
● B. Parinaud Syndrome
4. Limbic System
● A. Hippocampus
● B. Amygdala
● C. Kluver-Bucy Syndrome
Neurology: Diencephalon Bootcamp.com

• Hypothalamus:
• Homeostasis à regulating appetite, temperature, circadian rhythm, and growth
• Thalamus:
• Ventral lateral thalamic nucleus: Inà Cerebellum, basal ganglia : Out à Primary motor cortex
• Ventral posterolateral nucleus (VPL): Inà Spinothalamic tract, DCML : Out à Somatosensory cortex
• Ventral posteromedial nucleus (VPM): In à Trigeminal nerve, taste sensation : Out à Somatosensory cortex
• Lateral geniculate nucleus (LGN): In à Optic pathway, superior colliculi : Out à Primary visual cortex
• Medial geniculate nucleus (MGN): In à Inferior colliculi : Out à Primary auditory cortex
• Dorsomedial nucleus: In à Substantia nigra, amygdala, temporal cortex : Out à Prefrontal cortex
• Pineal Gland:
• Receives input from suprachiasmatic nucleus
• Regulation of sleep wake cycle via circadian release of melatonin
• Limbic System:
• Hippocampus: Memory formation, affected in Alzheimer’s dementia and early in hypoxia
• Amygdala: Emotion, fear, aggression
Neurology: Diencephalon Bootcamp.com

Hypothalamic Nuclei
Function Lesion Regulation
Anterior + Parasympathetics
Heat loss Hyperthermia
- Sympathetics

Posterior Heat retention Hypothermia + Sympathetics

Lateral + Ghrelin
Hunger Poor appetite
- Leptin

Ventromedial Satiety Hyperphagia + Leptin

Function Lesion
Arcuate Dopamine secretion ↑ prolactin

Preoptic Kallman syndrome


GnRH secretion
↓ GnRH

Paraventricular Oxytocin and ADH secretion


CRH, TRH release
Somatostatin release Central diabetes insipidus

Supraoptic Oxytocin and ADH secretion

Suprachiasmatic Circadian rhythm


Regulation of melatonin release Insomnia
(pineal gland)

Mamillary Body
Episodic memory Wernicke Encephalopathy
Neurology: Diencephalon Bootcamp.com

• Jet Lag Disorder:


• Suprachiasmatic nucleus dyssynchrony
• Kallman Syndrome (Hypogonadotropic hypogonadism):
• Anosmia à impaired migration of GnRH-neurons
• ↓ GnRH à ↓ FSH, ↓ LH
• Males: cryptorchidism, females: primary amenorrhea
• Wernicke Encephalopathy:
• History of chronic alcohol use
• Vitamin B1 (thiamine) deficiency à Avoid dextrose without thiamine supplementation
• Acute and reversible
• Ataxia, oculomotor disease, and encephalopathy
• Mamillary body (atrophy or
• Korsakoff Syndrome:
• Chronic Vitamin B1 deficiency
• Vitamin B1 (thiamine deficiency)
• Chronic and irreversible
• Confabulation with poor insight, anterograde (and/or retrograde) amnesia, personality changes
• Attention, cognition, and long-term memory generally preserved
Neurology: Diencephalon Bootcamp.com

• Heat exhaustion:
• T less than or equal to 104 F (40 C)
• No deficits in central neurologic functioning
• Nonexertional Heatstroke:
• T greater than 104 F (40 C)
• Bimodal distribution (children and elderly)
• Deficits in central neurologic functioning
• +/- absence of diaphoresis
• Exertional Heatstroke:
• T greater than 104 F (40 C).
• Healthy adults
• Deficits in central neurologic functioning
Neurology: Diencephalon Bootcamp.com

https://commons.wikimedia.org/wiki/File:Osborn_wave.gif

• Mild hypothermia:
• T 90-95 F (32-35 C)
• Tachycardia, tachypnea, possibly altered mental status
• Moderate hypothermia:
• T 82-90 F (28-32 C)
• Bradycardia, irregular breathing, possibly lethargy and general nervous system
depression
• Severe hypothermia:
• T less than 82 F (28 C)
• Hypotension, pulmonary edema, apnea, ventricular fibrillation
Neurology: Diencephalon Bootcamp.com

• Thalamic Syndrome:
• Severe contralateral burning, sharp pain affecting regions involved by previous stroke
• Lacunar Stroke Affecting the Thalamus:
• Pure Sensory à Ventroposterolateral and/or ventroposteriomedial thalamus (common)
• Contralateral sensory loss of face, upper and lower extremities (affecting multiple pathways)
Neurology: Diencephalon Bootcamp.com

https://commons.wikimedia.org/wiki/File:Tumor_Pineocytoma1.JPG

• Kluver-Bucy Syndrome:
• Hyperorality, hyperphagia, and hypersexuality +/- HSV encephalitis
• Parinaud Syndrome:
• Vertical gaze palsy, pseudo-Argyll Robertson pupils, convergence-retraction nystagmus
• Pinealoma may cause compression of cerebral aqueduct (obstructive hydrocephalus)
• Precocious puberty may be observed in setting of pinealoma
≣ ⟽ ⟾
Item 1 of 1 Test Your Knowledge
◽" Mark Difficulty: ✪✪ Bootcamp.com
Question ID: 0023 Previous Next

A 55-year-old male with no known past medical history presents to the emergency department by paramedics after being found
unconscious for an undetermined length of time. His initial glucose in the field was 28 mg/dL. The patient is managed acutely and
admitted to the hospital. The following day a more formal history is attempted, however the patient repeatedly interrupts the
physician stating, “everything looks blurry”. He appears to be generally disinterested in the conversation and displays
inattentiveness throughout the interview. No tremors are noted. His BMI is approximately 15 kg/m2 with otherwise normal vital
signs. Physical exam reveals horizontal nystagmus and significant postural ataxia. MRI of the brain and EEG are performed. EEG
is unremarkable. Previous medical records reveal that he was admitted one month prior for an episode of acute pancreatitis and
had previously reported living in a homeless shelter for the past year. During his last admission, there is no mention of
disorientation or confusion.

Which of the following findings on MRI would be most consistent with this patient’s presentation?

⚪ A. Diffuse atrophy of the frontal and temporal lobes bilaterally


⚪ B. Prominent cerebral sulci and ventriculomegaly
⚪ C. Pronounced atrophy of the caudate nucleus and ventriculomegaly
⚪ D. Pronounced atrophy of the mammillary bodies
⚪ E. Hyperintensity throughout the basal ganglia, temporal, and frontal cortices
≣ ⟽ ⟾
Item 1 of 1 Test Your Knowledge
◽" Mark Difficulty: ✪✪ Bootcamp.com
Question ID: 0023 Previous Next

A 55-year-old male with no known past medical history presents to the emergency department by paramedics after being found
unconscious for an undetermined length of time. His initial glucose in the field was 28 mg/dL. The patient is managed acutely and
admitted to the hospital. The following day a more formal history is attempted, however the patient repeatedly interrupts the
physician stating, “everything looks blurry”. He appears to be generally disinterested in the conversation and displays
inattentiveness throughout the interview. No tremors are noted. His BMI is approximately 15 kg/m2 with otherwise normal vital
signs. Physical exam reveals horizontal nystagmus and significant postural ataxia. MRI of the brain and EEG are performed. EEG
is unremarkable. Previous medical records reveal that he was admitted one month prior for an episode of acute pancreatitis and
had previously reported living in a homeless shelter for the past year. During his last admission, there is no mention of
disorientation or confusion.

Which of the following findings on MRI would be most consistent with this patient’s presentation?

⚪ A. Diffuse atrophy of the frontal and temporal lobes bilaterally


⚪ B. Prominent cerebral sulci and ventriculomegaly
⚪ C. Pronounced atrophy of the caudate nucleus and ventriculomegaly
" D. Pronounced atrophy of the mammillary bodies
⚪ E. Hyperintensity throughout the basal ganglia, temporal, and frontal cortices
OUTLINE

1. Anatomical Considerations

Neurology: ●

A. Substantia Nigra
B. Caudate Nucleus
● C. Putamen

Basal Ganglia ●

D. Globus Pallidus
E. Lentiform Nucleus
● F. Striatum
2. Basal Ganglia Physiologic Circuits
● A. Direct Pathway
● B. Indirect Pathway
3. Disorders of the Basal Ganglia
● A. Huntington Disease
● B. Wilson Disease
● C. Hemiballismus
● D. Parkinson Disease
● E. Dystonia
Neurology: Basal Ganglia Bootcamp.com
https://commons.wikimedia.org/wiki/File:Human_brain_frontal_(coronal)_section_description_2.JPG
https://commons.wikimedia.org/wiki/File:Brain_human_coronal_section.svg

• Caudate nucleus
• Putamen
• Globus pallidus
• Lentiform nucleus
• Striatum
Neurology: Basal Ganglia Bootcamp.com

https://commons.wikimedia.org/wiki/File:Basal_ganglia_circuits.svg

• Direct pathway à excitatory


• Indirect pathway à inhibitory
• Substantia nigra pars compacta à dopamine à net result ↑ motor activity
• Substantia nigra pars reticularis à GABA à net result ↓ motor activity
• Subthalamic nucleus à glutamate à net result ↓ motor activity

Substantia nigra pars compacta


D1 (+) striatum: direct pathway
D2 (-) striatum: indirect pathway
Both net result ↑ motor activity
Neurology: Basal Ganglia Bootcamp.com

https://radiopaedia.org/articles/huntington-disease

• Huntington Disease:
• Trinucleotide repeat expansion, CAG on huntingtin (HTT) gene on chromosome 4
• Autosomal dominant, anticipation, gain of function mutation
• Caudate nucleus atrophy, ventriculomegaly
• Chorea (hyperkinetic) à Bradykinesia (hypokinetic)
• Late stage: Aggressive behavior, dementia, psychosis
• Wilson Disease:
• Hepatolenticular degeneration (atrophy of lentiform nucleus)
• Hepatic disease, neuropsychiatric changes
• Kayser-Fleischer rings (slit-lamp)
• Wing-beating tremor
• Hemiballismus:
• Contralateral subthalamic nucleus lesion
• Involuntary, flinging movements
• Parkinson Disease:
• Substantia nigra degeneration
• Bradykinesia, rigidity, resting tremor
• Dystonia:
• Cervical dystonia (spasmodic torticollis)
• Blepharospasm (uncontrollable blinking à closure of eyelids)
• Acute iatrogenic dystonia (neuroleptics)
≣ ⟽ ⟾
Item 1 of 1 Test Your Knowledge
◽" Mark Difficulty: ✪✪ Bootcamp.com
Question ID: 0024 Previous Next https://commons.wikimedia.org/wiki/File:Human_brain_frontal_(coronal)_section_description_2.JPG

A 41-year-old female presents to her primary care


physician for worsening memory loss. She states that she
is concerned she may have Alzheimer’s dementia. Last
week she states that she forgot her car keys in her vehicle.
She reports that she had a fall episode recently as well, at
which time she simply lost her balance while cooking A
dinner. She feels as if her arms “have a mind of their own”.
She is noted to demonstrate involuntary, irregular
movements of the upper limbs and writhing movements of
the fingers. Nystagmus is also noted when performing B
testing of the extraocular muscles. L4 reflex is 3+
bilaterally. She is unaware of any family history of disease
as she was adopted at a very young age. Which of the
following structures is most likely to be affected given the
patient’s presentation?
C
⚪ A.
⚪ B. D
⚪ C. E
⚪ D.
⚪ E.
≣ ⟽ ⟾
Item 1 of 1 Test Your Knowledge
◽" Mark Difficulty: ✪✪ Bootcamp.com
Question ID: 0024 Previous Next https://commons.wikimedia.org/wiki/File:Human_brain_frontal_(coronal)_section_description_2.JPG

A 41-year-old female presents to her primary care


physician for worsening memory loss. She states that she
is concerned she may have Alzheimer’s dementia. Last
week she states that she forgot her car keys in her vehicle.
She reports that she had a fall episode recently as well, at
which time she simply lost her balance while cooking A
dinner. She feels as if her arms “have a mind of their own”.
She is noted to demonstrate involuntary, irregular
movements of the upper limbs and writhing movements of
the fingers. Nystagmus is also noted when performing B
testing of the extraocular muscles. L4 reflex is 3+
bilaterally. She is unaware of any family history of disease
as she was adopted at a very young age. Which of the
following structures is most likely to be affected given the
patient’s presentation?
C
" A.
⚪ B. D
⚪ C. E
⚪ D.
⚪ E.
OUTLINE

1. Dopaminergic Pathways

Neurology: ●

A. Mesocortical
B. Mesolimbic
● C. Tuberoinfundibular

Neurotransmitter ●
2.
D. Nigrostriatal
Neurotransmitters in Psychiatric Disease

Activity in ●


A. Acetylcholine
B. Serotonin
C. Dopamine

Psychiatric ●

D. Norepinephrine
E. GABA

Disease
Neurology: Neurotransmitter Activity in Psychiatric Disease Bootcamp.com

https://commons.wikimedia.org/wiki/File:Dopaminergic_pathways.svg

• Mesolimbic:
• Lesion à Positive symptoms of schizophrenia
• Mesocortical:
• Lesion à Negative symptoms of schizophrenia
• Tuberoinfundibular:
• Lesion à ↓ dopamine à ↑ prolactin
• Nigrostriatal:
• Inhibition à Parkinsonism, extrapyramidal symptoms
• Stimulation à Chorea
Neurology: Neurotransmitter Activity in Psychiatric Disease Bootcamp.com

• Nucleus Basalis of Meynert:


• Acetylcholine: ↑ Parkinson Disease : ↓ Alzheimer’s Dementia, Huntington's Disease
• Substantia Nigra pars compacta, Arcuate nucleus:
• Dopamine: ↑ Huntington disease, Schizophrenia (+ symptoms) : ↓ Parkinson Disease
• Locus Coeruleus:
• Norepinephrine: ↑ Anxiety : ↓ Depression
• Raphe Nucleus:
• Serotonin: ↑ Schizophrenia : ↓ Depression, Anxiety
• Nucleus Accumbens:
• GABA: ↓ Huntington’s Disease, Anxiety

Schizophrenia
Negative s/s: ↓ dopamine in Mesocortical pathway
Positive s/s: ↑ dopamine in Mesolimbic pathway
≣ ⟽ ⟾
Item 1 of 1 Test Your Knowledge
◽" Mark Difficulty: ✪✪ Bootcamp.com
Question ID: 0022 Previous Next

A 26-year-old male is seen by his primary care physician with his spouse
for an annual physical. His spouse reports that he began to demonstrate
peculiar behavior approximately two months earlier that has now become
increasingly concerning. She states that he had been stressed out from
working more hours than usual. Initially, he was having difficulty sleeping
and became paranoid of those around him. He repeatedly tried to
convince her that he was receiving messages from a secret underground
location and that he was engaged in intermittent discussions with a spy
from another country. She states that he has recently begun talking to
himself when alone. During the encounter he continues to repeat select
phrases used by the physician and displays chaotic tangential speech.

Which of the following is most consistent with the pattern of


neurotransmitter activity in this patient’s central nervous system?

⚪ A. ↑ Norepinephrine, ↓ Dopamine
⚪ B. ↑ Norepinephrine, ↓ Serotonin
⚪ C. ↑ GABA, ↓ Dopamine
⚪ D. ↑ GABA, ↑ Serotonin
⚪ E. ↑ Serotonin, ↑ Dopamine
≣ ⟽ ⟾
Item 1 of 1 Test Your Knowledge
◽" Mark Difficulty: ✪✪ Bootcamp.com
Question ID: 0025 Previous Next

A 26-year-old male is seen by his primary care physician with his spouse
for an annual physical. His spouse reports that he began to demonstrate
peculiar behavior approximately two months earlier that has now become
increasingly concerning. She states that he had been stressed out from
working more hours than usual. Initially, he was having difficulty sleeping
and became paranoid of those around him. He repeatedly tried to
convince her that he was receiving messages from a secret underground
location and that he was engaged in intermittent discussions with a spy
from another country. She states that he has recently begun talking to
himself when alone. During the encounter he continues to repeat select
phrases used by the physician and displays chaotic tangential speech.

Which of the following is most consistent with the pattern of


neurotransmitter activity in this patient’s central nervous system?

⚪ A. ↑ Norepinephrine, ↓ Dopamine
⚪ B. ↑ Norepinephrine, ↓ Serotonin
⚪ C. ↑ GABA, ↓ Dopamine
⚪ D. ↑ GABA, ↑ Serotonin
$ E. ↑ Serotonin, ↑ Dopamine
OUTLINE

1. Dementia Differential
Neurology: ●

A. Alzheimer Dementia
B. Parkinson Disease

Dementia
● C. Vascular Dementia
● D. Frontotemporal Dementia
● E. Lewy Body Dementia
● F. Creutzfeldt-Jakob Disease
● G. Normal Pressure Hydrocephalus
● H. Hypothyroidism
● I. Major Depressive Disorder
● J. Neurosyphilis
● K. Vitamin B12 Deficiency
● L. Huntington Disease
● M. HIV-Associated Dementia
Neurology: Dementia Bootcamp.com

https://commons.wikimedia.org/wiki/File:Cerebral_amyloid_angiopathy_-2a-_amyloid_beta_-_high_mag.jpg
https://upload.wikimedia.org/wikipedia/commons/3/3c/MRI_Location_Hippocampus_up..png
https://commons.wikimedia.org/wiki/File:Tauopathy_in_Alzheimer%27s_disease.jpg
https://upload.wikimedia.org/wikipedia/commons/a/af/Amyloidosis%2C_lymph_node%2C_polarizer.jpg

• Pathophysiology:
• Amyloid precursor protein (APP) gene on chromosome 21
• β-amyloid plaques (Aβ)à neurotoxic
• ↓ ACh (nucleus basalis of Meynert)
• Histopathology:
• Senile plaques (Aβ + dystrophic neurites, extracellular)
• Stain positive with Congo red (apple-green birefringence)
• Neurofibrillary tangles (hyperphosphorylated tau protein, intracellular)
• Amyloid angiopathy
• Presentation:
• Primary Early Finding: Progressively worsening short term memory loss
• Other findings: Executive dysfunction; visuospatial, behavioral and language impairment
• Imaging:
• Disproportionate hippocampal and/or temporoparietal lobe atrophy
• Generalized cortical atrophy +/- hydrocephalus ex vacuo
• Associations:
• Cerebral amyloid angiopathy
• Management:
• Acetylcholinesterase inhibitors à Donepezil, Rivastigmine, Galantamine
• NMDA receptor antagonist à Memantine
Neurology: Dementia Bootcamp.com

MPTP Consumption:
-Metabolite causes Parkinsonism
• Pathophysiology: -MPTP metabolized via MAO-B
• Dopaminergic depletion in substantia nigra pars compacta -Rx: Selegiline
• ↑ Stimulation to Gpi à ↓ net motor movement
• ↑ Acetylcholine, ↓ Serotonin and norepinephrine Iatrogenic
• Histopathology: -Typical antipsychotics: Haloperidol
• Intracellular eosinophilic inclusions (!-synuclein) = Lewy bodies -Metoclopramide
• Predominantly observed in substantia nigra and locus coeruleus
• Presentation:
• Bradykinesia, rigidity (Cogwheel), resting tremor
• Imbalance and postural instability
• Shuffling gait
• Management:
• Dopamine analog à Levodopa
• DOPA-decarboxylase (peripheral) inhibitor à Carbidopa
• Catechol-O-methyltransferase (COMT) inhibitors à Entacapone and tolcapone
• Dopamine agonist à Bromocriptine, pramipexole, and ropinirole
• Monoamine oxidase-B (MAO-B) inhibitor à Selegiline
• Anticholinergic à Benztropine and trihexyphenidyl
• Mix of targets à Amantadine
Neurology: Dementia Bootcamp.com

• Pathophysiology:
• Recurrent infarctions and cerebral ischemia
• Histopathology:
• Glial scar formation, cavitary lesions
• Presentation:
• Primary Early Finding: Step-wise decline, executive dysfunction
• Other findings: Memory impairment, behavioral changes
• Asymmetric sensory or motor defects, homonymous hemianopia or quadrantopia, upper motor neuron signs
• Imaging:
• Multiple cortical and/or lacunar infarcts with white matter lesions
• +/- hydrocephalus ex vacuo
• Associations:
• Small vessel à Significant cardiovascular and cerebrovascular risk factors (hypertension, hyperlipidemia, diabetes)
• Large vessel à Atrial fibrillation, patent foramen ovale
Neurology: Dementia Bootcamp.com
https://commons.wikimedia.org/wiki/File:Histology_of_frontotemporal_lobar_degeneration.jpg
https://commons.wikimedia.org/wiki/File:Pick%27s_disease.png

• Histopathology:
• Cytoplasmic circular inclusions (hyperphosphorylated tau protein, Pick bodies)
• Presentation:
• Primary Early Finding: Personality and behavioral changes
• Other findings: Disinhibition, hyperorality, compulsive behavior
• Memory deficits are minimal
• Imaging:
• Frontotemporal atrophy
Neurology: Dementia Bootcamp.com

https://commons.wikimedia.org/wiki/File:Lewy_bodies_(alpha_synuclein_inclusions).svg

Parkinson Disease vs Lewy Body Dementia


Parkinson Disease: Motor THEN Cognitive
• Histopathology: Lewy Body Dementia: Motor AND Cognitive
• Intracellular eosinophilic inclusions (!-synuclein) = Lewy bodies
• Located diffusely throughout the brain
• Presentation:
• Primary Early Finding: Fluctuating cognition, Parkinsonism, visual hallucinations
• Other Findings: Visuospatial and executive impairment
• Associations:
• Increased sensitivity to antipsychotics

REM Sleep Behavior Disorder


↓ REM sleep atonia à Dream enactment
Possible early marker of PD or LBD
Neurology: Dementia Bootcamp.com

• Pathophysiology:
• PrPc = normal structure conformation of prion protein
• PrPSC = abnormal structure resistant to enzymatic breakdown
• PrPSC can convert PrPc à PrPSC
• Histopathology:
• Large intracytoplasmic vacuoles (spongiform)
• Presentation:
• Primary Early Finding: Rapidly progressive dementia + myoclonus
• Diagnostics:
• ↑ 14-3-3 protein
• EEG: Triphasic periodic sharp waves
• Associations:
• Iatrogenic (corneal or dural graft transplant)
Neurology: Dementia Bootcamp.com

• Normal Pressure Hydrocephalus:


• Classic Triad: Gait ataxia (“magnetic gait”), urinary incontinence, dementia
• Ventriculomegaly in or out of proportion to cortical atrophy (vs Hydrocephalus ex vacuo)
• NPH: Ventriculomegaly tends to be the prominent finding
• Hydrocephalus ex vacuo: Sulci enlargement and cortical atrophy tend to be the prominent findings
• Symptomatic improvement with high-volume lumbar puncture and/or VP shunt
• Major Depressive Disorder:
Reversible Causes of Dementia
• Pseudodementia Major depressive disorder
• Normal neurologic evaluation and neuroimaging Thyroid disease
• Vitamin B12 Deficiency: Neurosyphilis
Vitamin B12 deficiency
• Peripheral neuropathy, subacute combined degeneration
• Huntington Disease:
• Trinucleotide repeat expansion, CAG on huntingtin (HTT) gene on chromosome 4
• Autosomal dominant, anticipation, gain of function mutation
• Caudate nucleus atrophy, ventriculomegaly
• Chorea (hyperkinetic) à Bradykinesia (hypokinetic)
• Late stage: Aggressive behavior, dementia, psychosis
• HIV-Associated Dementia:
• More common in severe disease (AIDS, CD4<200)
• Progressive cognitive decline
• Target: Microglial cells (phagocytes of CNS)
• Multinucleated giant cells and microglial nodules
≣ ⟽ ⟾
Item 1 of 1 Test Your Knowledge
◽" Mark Difficulty: ✪✪ Bootcamp.com
Question ID: 0026 Previous Next

A 48-year-old female employed as an analytical chemist with no past


medical history is seen in the emergency department for chemical-
induced keratitis. She is emergently seen by an ophthalmologist who
performs a corneal graft transplantation of the affected eye. She tolerates
the procedure well and is subsequently discharged from the hospital. Two
weeks later she returns to the emergency department with her son who
reports that she has been behaving erratically. She demonstrates
involuntary, jerking movements of all four extremities that are worsened
by loud noises. A CT scan of the head and MRI are relatively unrevealing.
Lumbar puncture is performed revealing an increase in S100 and 14-3-3
protein.

Which of the following would be most consistent with the findings


observed on a brain specimen of this patient?

⚪ A. Diffuse extracellular neuritic plaques scattered throughout the gray matter


⚪ B. Intracellular rod-shaped eosinophilic aggregates in the hippocampus
⚪ C. Intracytoplasmic vacuoles within neurons of the cerebral cortex
⚪ D. Liquefactive necrosis and cystic cavity formation localized to the internal capsule
⚪ E. Significant depletion of dopaminergic neurons within the substantia nigra pars compacta
≣ ⟽ ⟾
Item 1 of 1 Test Your Knowledge
◽" Mark Difficulty: ✪✪ Bootcamp.com
Question ID: 0026 Previous Next

A 48-year-old female employed as an analytical chemist with no past


medical history is seen in the emergency department for chemical-
induced keratitis. She is emergently seen by an ophthalmologist who
performs a corneal graft transplantation of the affected eye. She tolerates
the procedure well and is subsequently discharged from the hospital. Two
weeks later she returns to the emergency department with her son who
reports that she has been behaving erratically. She demonstrates
involuntary, jerking movements of all four extremities that are worsened
by loud noises. A CT scan of the head and MRI are relatively unrevealing.
Lumbar puncture is performed revealing an increase in S100 and 14-3-3
protein.

Which of the following would be most consistent with the findings


observed on a brain specimen of this patient?

⚪ A. Diffuse extracellular neuritic plaques scattered throughout the gray matter


⚪ B. Intracellular rod-shaped eosinophilic aggregates in the hippocampus
$ C. Intracytoplasmic vacuoles within neurons of the cerebral cortex
⚪ D. Liquefactive necrosis and cystic cavity formation localized to the internal capsule
⚪ E. Significant depletion of dopaminergic neurons within the substantia nigra pars compacta
OUTLINE

1. Headache Differential
Neurology: ●

A. Tension Headache
B. Migraine

Headache
● C. Cluster Headache
● D. Idiopathic Intracranial Hypertension
● E. Trigeminal Neuralgia
Neurology: Headache Bootcamp.com

• Presentation:
• Band-like, bilateral “achy, tight, or dull”, “vice-like” headache
• Classically bifrontal
• Musculoskeletal tenderness
• No neurologic symptoms
• Duration: Variable
• Triggers:
• Stress #1, lack of sleep, anxiety
• Associations:
• Women > Men
• Generally, no genetic predisposition
• Management:
• NSAIDs and/or conservative
Neurology: Headache Bootcamp.com

https://commons.wikimedia.org/wiki/File:ScintillatingScotoma3.jpg

• Pathophysiology
• ↑ CNS excitability à abnormal activation of trigeminal fibers à ↑ CGRP
• Calcitonin gene-related peptide: Vasodilation and inflammatory response
• Presentation:
• Unilateral, ”throbbing, pulsating” headache
• +/- Aura à reversible neurologic symptoms
• +/- Photophobia, nausea, emesis
• Duration: 4-72 hours Migraine POUND:
Pulsatile
• Triggers: One day of duration
• Alcohol, nicotine, stress, poor sleeping habits
Unilateral
• Associations:
• Women > Men
Nausea, emesis
• Genetic predisposition Debilitating
• Acute Management:
• Sumatriptan: 5-HT1B/1D agonist à Inhibition of CGRP, vasoconstriction
• Ergotamine, Dihydroergotamine: 5-HT1B/1D activity + !-adrenergic activity à vasoconstriction
Neurology: Headache Bootcamp.com

• Presentation:
• Severe acute onset, “sharp” unilateral periorbital and temporal headache
• Ipsilateral autonomic symptoms (nasal congestion, lacrimation, rhinorrhea, miosis)
• No aura
• Duration: <3 hours, multiple attacks per day
• Repeating in “clusters”, cyclical association between attacks
• Triggers:
• Clustered in similar times daily, +/- during sleep
• Associations:
• Men > Women
• Tobacco use
• +/- Genetic predisposition
• Acute Management:
• Oxygen, 100% FiO2
• Sumatriptan: 5-HT1B/1D agonist à Inhibition of CGRP, vasoconstriction
Neurology: Headache Bootcamp.com
https://commons.wikimedia.org/wiki/File:Fundus_photograph_of_normal_left_eye.jpg
https://commons.wikimedia.org/wiki/File:Papilledema.jpg

• Presentation:
• Young, obese female
• Frequent non-localized headaches
• Signs of ↑ ICP à Bilateral symmetric papilledema, lateral rectus palsy
• Pulsatile tinnitus
• Transient visual disturbances and/or diplopia
• Associations:
• Women > Men
• Vitamin A
• Management:
• MRI à Generally unremarkable
• LP à ↑ opening pressure, diagnostic and therapeutic
• Acetazolamide: Carbonic anhydrase inhibitor
Neurology: Headache Bootcamp.com

• Presentation:
• Older patient (55-70 years of age)
• Unilateral, severe, sudden onset, “shooting or stabbing”, “electric shock” pain
• V2 and V3 distributions classically affected
• Duration: Several seconds, recurring many times throughout the day
• Triggers:
• Talking, chewing, touch
• Management:
• MRI à Generally unremarkable
• Carbamazepine: Inactivation of depolarizing Na+ channels
Neurology: Headache Bootcamp.com

High Yield Headache


Disease Process Classic Presentation Duration Neurologic Key Associations Acute Management Preventative/Chronic
Symptoms Management

Tension Headache Band-like Variable None, no aura n/a NSAIDs and/or Avoid precipitating
Bilateral conservative factors
“Achy”, “tight”, or “dull”
“Vice-like” headache

Migraine “Throbbing, pulsating” 4-72 hours +/- Aura Women Sumatriptan β-blocker
Unilateral Photophobia Genetic predisposition Ergotamine TCA
Nausea Topiramate
(low yield)

Cluster Headache ”Sharp” unilateral, periorbital <3 hours No aura Men Oxygen 100% FiO2 Verapamil (low yield)
Repeating in “clusters” Ipsilateral autonomic Tobacco use Sumatriptan
Cyclical association between symptoms
attacks (similar times daily)

Idiopathic Non-localized Variable Pulsatile tinnitus Women (Young, obese) Therapeutic high n/a
Intracranial Signs of ↑ ICP (ex. Papilledema) Diplopia (ex. Lateral Vitamin A volume lumbar
Hypertension rectus palsy) puncture
Acetazolamide

Trigeminal Neuralgia Sudden onset Seconds n/a > Age (55*-70) n/a to Step 1 Carbamazepine
Localized to V2 and V3
distribution
“Electric”, “shock-like”
“Shooting” or “stabbing”
Worse with talking, chewing, and
shaving

Keep in mind: Test writers may use adjectives such as “sharp” to describe a migraine headache in a
question stem. Classic descriptions are not pathognomonic.
≣ ⟽ ⟾
Item 1 of 1 Test Your Knowledge
◽" Mark Difficulty: ✪✪ Bootcamp.com
Question ID: 0027 Previous Next
https://commons.wikimedia.org/wiki/File:Fundus_photograph_of_normal_right_eye.jpg

A 34-year-old female with a past medical history of generalized anxiety disorder presents to the emergency department for a headache
that has been ongoing for the past 2 hours. The patient reports that she was in a meeting at her office when she suddenly saw “stars” and
felt her left arm go numb. She initially thought she may be having a stroke, however her symptoms resolved within five minutes. Shortly
thereafter she developed a left-sided 8/10 throbbing headache. She has been unable to eat due to significant nausea. She requests that
the lights remain dimmed in the room. Her grandmother has a history of recurrent headaches with no other significant family history. Oral
contraceptives are the only prescribed medication that she is currently taking. Vital signs and fundoscopy are shown below. Physical
examination is unremarkable.

Temperature: 98F (36.7C)


Blood Pressure: 130/62
Heart Rate: 104/min
Respiratory Rate: 12/min
Oxygen Saturation: 100% on room air

Non-contrast head CT is unremarkable. A serotonin agonist is prescribed to acutely manage the severity of this patient’s headache. Which
mechanism of action is most likely consistent with the medication prescribed?

⚪ A. Inhibition of enzymatic carbonic anhydrase


⚪ B. Binding to inactive voltage-gated sodium channels in cortical tissue
⚪ C. Inhibition of calcitonin gene-related peptide release from trigeminal afferent neurons
⚪ D. Stimulation of trigeminal afferent neuronal fibers to release substance P
⚪ E. Intracranial vessel vasodilation secondary to binding of target receptors on smooth muscle tissue
≣ ⟽ ⟾
Item 1 of 1 Test Your Knowledge
◽" Mark Difficulty: ✪✪ Bootcamp.com
Question ID: 0027 Previous Next
https://commons.wikimedia.org/wiki/File:Fundus_photograph_of_normal_right_eye.jpg

A 34-year-old female with a past medical history of generalized anxiety disorder presents to the emergency department for a headache
that has been ongoing for the past 2 hours. The patient reports that she was in a meeting at her office when she suddenly saw “stars” and
felt her left arm go numb. She initially thought she may be having a stroke, however her symptoms resolved within five minutes. Shortly
thereafter she developed a left-sided 8/10 throbbing headache. She has been unable to eat due to significant nausea. She requests that
the lights remain dimmed in the room. Her grandmother has a history of recurrent headaches with no other significant family history. Oral
contraceptives are the only prescribed medication that she is currently taking. Vital signs and fundoscopy are shown below. Physical
examination is unremarkable.

Temperature: 98F (36.7C)


Blood Pressure: 130/62
Heart Rate: 104/min
Respiratory Rate: 12/min
Oxygen Saturation: 100% on room air

Non-contrast head CT is unremarkable. A serotonin agonist is prescribed to acutely manage the severity of this patient’s headache. Which
mechanism of action is most likely consistent with the medication prescribed?

⚪ A. Inhibition of enzymatic carbonic anhydrase


⚪ B. Binding to inactive voltage-gated sodium channels in cortical tissue
$ C. Inhibition of calcitonin gene-related peptide release from trigeminal afferent neurons
⚪ D. Stimulation of trigeminal afferent neuronal fibers to release substance P
⚪ E. Intracranial vessel vasodilation secondary to binding of target receptors on smooth muscle tissue
OUTLINE

1. Overview

Neurology: ●

A. Classification
B. Epilepsy
● C. General Terms

Seizures ●
2.
D. Triggers and Inciting Causes
Focal Seizure
● A. Focal Aware
● B. Focal Impaired Awareness
3. Generalized Onset Seizure
● A. Absence
● B. Tonic-Clonic
● C. Myoclonic
● D. Atonic
● E. Simple Febrile
4. Status Epilepticus
● A. Definition
● B. Management
5. Antiepileptic Medications
● A. Narrow Spectrum
● B. Broad Spectrum
● C. Other/Varied
Neurology: Seizure Bootcamp.com

• Seizure Classification:
• Unprovoked: No clear etiology
• Reflex: Trigger that predisposes to seizure risk (↓ seizure threshold)
• Provoked: Concurrent with systemic illness or central nervous system pathology
• Epilepsy:
• ≥2 unprovoked separated by >24 hours
• 1 seizure with high risk for subsequent seizure
• General Terms:
• Ictal: Occurring during the time of seizure
• Post-ictal: Occurring after the time of seizure
• Triggers à Reflex:
• Flashing lights
• High fever (infants and young children)
• Lack of sleep
• Inciting Causes à Provoked:
• Cerebrovascular accident
• Traumatic brain injury
• Electrolyte imbalance
Neurology: Seizure Bootcamp.com

Frontal: Parietal:
Jerking movement Numbness of
of contralateral contralateral
• Origin: extremity extremity
• Single hemisphere (à +/- global generalization)
• Localized symptoms at onset
Temporal: Occipital:
• Focal Aware (Simple Partial):
Auditory Visual
• Awareness and consciousness remains intact
hallucinations hallucinations
• No post-ictal state
• Focal Impaired Awareness (Complex Partial):
• Awareness impaired, ”blank stare” Jacksonian March:
• Post-ictal state (common) à Todd paralysis Spread of abnormal electrical
activity to nearby areas of the
• Automatisms motor cortex
• Common origin à Temporal lobe
• Etiology:
• #1 Focal cerebral lesion Focal Seizure Presentation à Contralateral Face or Limb
Automatisms
Myoclonus
Tonic and/or clonic Contractions
Sensory disturbances
Behavioral changes
Neurology: Seizure Bootcamp.com

• Origin:
• Global generalization
• Generalized symptoms at onset
• Absence:
• Loss of consciousness, “blank stare” for short intervals (~ 5-15 seconds)
• Generally, no post-ictal state, consciousness returns immediately following ictal-state
• Child or adolescent
• EEG à 3-Hz spike-wave complex
• First-line Rx: Ethosuximide
• Tonic-Clonic (grand mal):
• Loss of consciousness
• Post-ictal state (common)
• Bilateral muscle contractions (tonic) and rhythmic twitching (clonic)
• “Eyes rotate to the back of the head”
• Lateral tongue lacerations
• Urinary or stool incontinence
• First-line Rx: Broad-spectrum AEDs (e.g., Levetiracetam)
• Myoclonic:
• Nonrhythmic jerking movements
• No loss of consciousness, no post-ictal state
• Atonic:
• “Drop seizure”
• Acute onset diffuse loss of muscle tone
Neurology: Seizure Bootcamp.com

• Pathophysiology:
• Hyperthermia à ↑ CSF cytokines/interleukins
• Presentation:
• Significant fever (>104F)
• 6 months to 5 years of age
• Simple Febrile Seizure: Brief generalized, nonrecurring seizure
• Complex Febrile Seizure: Long lasting focal at onset, recurring seizure
• Associations:
• Viral infection: Classic cause = Roseola à HHV-6
• Genetic predisposition
• Management:
• Simple febrile seizure: Conservative
• NSAIDs, acetaminophen à ↓ PGE2 à ↓ Central hypothalamic temperature setpoint
Neurology: Seizure Bootcamp.com

• Definition:
• Single seizure, ≥5 minutes
• Multiple seizures, incomplete level of consciousness regained between each episode
• Management:
• Abort seizure: Intravenous benzodiazepines à GABAA agonists à hyperpolarizing
• Prevention of recurrence: Intravenous phenytoin
Neurology: Seizure Bootcamp.com

https://commons.wikimedia.org/wiki/File:Gingivitis_(crop).jpg

Fetal Hydantoin Syndrome


• Phenytoin: Fingernail hypoplasia
• MOA: Inhibition of pre-synaptic voltage-gated sodium channels Excess hair production
• Adverse effects: Teratogenic, gingival hyperplasia, hirsutism
• Cytochrome P450 inducer, zero order elimination
Intrauterine growth restriction

Narrow
• Carbamazepine:
• Primary use: Focal seizures and trigeminal neuralgia
• MOA: Inhibition of pre-synaptic voltage-gated sodium channels
• Cytochrome P450 inducer (strong)
• Adverse effects: SJS, Aplastic anemia, teratogenic
• Gabapentin: Inhibition of pre-synaptic voltage-gated calcium channels
• Ethosuximide: Inhibition of voltage-gated T-type calcium channels in thalamus

• Valproate: Adverse effects: Neural tube defects

Broad
• Levetiracetam: Binds SV2A à modulation of glutamate and GABA release
• Lamotrigine: Adverse effects: Stevens-Johnson syndrome

• Phenobarbital:
• Primary use: Neonatal seizure
• MOA: GABA-A agonist, ↑ duration of chloride channel opening

Other
• Cytochrome P450 inducer
• Lorazepam:
• Primary use: Abort acute seizure activity and status epilepticus
• MOA: GABA-A agonist, ↑ frequency of chloride channel opening
≣ ⟽ ⟾
Item 1 of 1 Test Your Knowledge
◽" Mark Difficulty: ✪✪ Bootcamp.com
Question ID: 0028 Previous Next
https://commons.wikimedia.org/wiki/File:Spike-waves.png

A 6-year-old female with no significant past medical history is seen by her pediatrician
for worsening academic performance. Her father is convinced that she has attention
deficit disorder. He references three occasions of which he was contacted by his
daughter’s teacher to inform him of her “day-dreaming”. He also states that she stares
off into the distance when playing with her friends outside. The patient has no history
of missed milestones and height, and weight are within the appropriate reference
ranges for her age. Her temperature is 98F (36.7C) with otherwise unremarkable vital
signs. On physical examination no focal neurologic deficits are appreciated. Tongue
lacerations are absent. Electroencephalography is performed in the setting of
hyperventilation. The results are shown below. A medication is prescribed by her
pediatrician.

Which mechanism of action is most likely consistent with the medication prescribed?

⚪ A. Reversible inhibition of cyclooxygenase-1 enzymes


⚪ B. Increase in frequency of chloride channel opening
⚪ C. Increase in duration of chloride channel opening
⚪ D. Inhibition of inactivated sodium channels
⚪ E. Selective norepinephrine reuptake inhibitor
⚪ F. Inhibition of thalamic T-type calcium channels
≣ ⟽ ⟾
Item 1 of 1 Test Your Knowledge
◽" Mark Difficulty: ✪✪ Bootcamp.com
Question ID: 0028 Previous Next
https://commons.wikimedia.org/wiki/File:Spike-waves.png

A 6-year-old female with no significant past medical history is seen by her pediatrician
for worsening academic performance. Her father is convinced that she has attention
deficit disorder. He references three occasions of which he was contacted by his
daughter’s teacher to inform him of her “day-dreaming”. He also states that she stares
off into the distance when playing with her friends outside. The patient has no history
of missed milestones and height, and weight are within the appropriate reference
ranges for her age. Her temperature is 98F (36.7C) with otherwise unremarkable vital
signs. On physical examination no focal neurologic deficits are appreciated. Tongue
lacerations are absent. Electroencephalography is performed in the setting of
hyperventilation. The results are shown below. A medication is prescribed by her
pediatrician.

Which mechanism of action is most likely consistent with the medication prescribed?

⚪ A. Reversible inhibition of cyclooxygenase-1 enzymes


⚪ B. Increase in frequency of chloride channel opening
⚪ C. Increase in duration of chloride channel opening
⚪ D. Inhibition of inactivated sodium channels
⚪ E. Selective norepinephrine reuptake inhibitor
$ F. Inhibition of thalamic T-type calcium channels
OUTLINE

1. Diffuse Axonal Injury

Neurology: ●

A. Etiology
B. Pathophysiology
● C. Histopathology

Traumatic Brain ●
2.
D. Imaging
Intracranial Hypertension

Injury and ●


A. Classic Etiologies
B. Cushing Reflex
C. Signs of Increased Intracranial Pressure

Herniation ●

D. Imaging Findings
E. Management
3. Brain Herniation
● A. Uncal (Transtentorial) Herniation
● B. Subfalcine (Cingulate) Herniation
● C. Tonsillar Herniation
4. Decerebrate vs Decorticate Posturing
● A. Decerebrate Posturing
● B. Decorticate Posturing
● C. Absence of Motor Tone
Neurology: Traumatic Brain Injury and Herniation Bootcamp.com
https://commons.wikimedia.org/wiki/File:Diffuse_axonal_injury-_cMRT_nach_3_Tagen.jpg
https://commons.wikimedia.org/wiki/File:Contrecoup.svg

• Etiology: Initial diagnostics in TBIà Head CT


• Coup-contrecoup injury, extreme rotational acceleration/deceleration forces
• Motor vehicle accidents
• Penetrating and blast trauma
• Pathophysiology:
• Widespread shearing of white matter tracts
• Histopathology:
• Axonal bulb formation (diffuse axonal edema)
• Accumulation of axonal transport proteins
• Imaging:
• Head CT scan: Generally unremarkable
• Brain MRI: Diffuse punctate hemorrhages at the site of the gray-white matter junction and corpus callosum
Neurology: Traumatic Brain Injury and Herniation Bootcamp.com

CPP = MAP - ICP


• Classic Etiologies:
• Mass lesion (brain tumor, aneurysm, hematoma)
• Idiopathic intracranial hypertension
• Hydrocephalus
• Intracranial hemorrhage
• Cushing Reflex:
• Hypertension (↑ MAP)
• Bradycardia
• Irregular respirations à Respiratory depression
• Signs of ↑ Intracranial Pressure
• Bilateral papilledema
• Lateral rectus palsy
• Ipsilateral fixed, dilated pupil, “down and out” eye +/- ptosis
• Bulging fontanelle (infants)
• Imaging Findings:
• Active hemorrhage, midline shift
• Ventriculomegaly
• Management:
• Hyperventilation (↓PaCO2 à vasoconstriction à ↓ cerebral blood volume)
• Elevate head of the bed
• Intravenous mannitol
Neurology: Traumatic Brain Injury and Herniation Bootcamp.com

https://commons.wikimedia.org/wiki/File:Brain_herniation_types-2.svg

• Uncal (Transtentorial) Herniation:


• Medial temporal lobe beneath tentorium cerebelli
• Initial Sign: Ipsilateral fixed, dilated pupil (CN III) à “down and out” positioning +/- ptosis
• Ipsilateral and/or contralateral weakness (cerebral peduncle)
• Contralateral homonymous hemianopia with macular sparing or blindness (PCA)
• Subfalcine (Cingulate) Herniation:
• Cingulate gyrus under falx cerebri
• Contralateral lower extremity weakness (ACA territory)
• Obstructive hydrocephalus (foramen of Monro)
• Tonsillar Herniation:
• Classic Cause: Chiari malformations
• Obstructive hydrocephalus (cerebral aqueduct)
Neurology: Traumatic Brain Injury and Herniation Bootcamp.com

https://commons.wikimedia.org/wiki/File:Decorticate.PNG
https://commons.wikimedia.org/wiki/File:Decerebrate.jpg

• Decorticate (Flexor) Posture:


• Lesion to neural tissue above the red nucleus
• ↑ neuronal activity to the upper limb flexors (Flexors > Extensors)
• Decerebrate (Extensor) Posture:
• Lesion to neural tissue below the red nucleus
• ↓ neuronal activity to the upper limb flexors (Flexors < Extensors)
• Absence of Motor Tone:
• Classic Cause: Cervical spinal cord injury
≣ ⟽ ⟾
Item 1 of 1 Test Your Knowledge
◽" Mark Difficulty: ✪✪ Bootcamp.com
Question ID: 0029 Previous Next
https://commons.wikimedia.org/wiki/File:Subfalcine-herniation-001.jpg

A 67-year-old female is brought to the emergency department by paramedics following


acute onset right-sided lower extremity weakness and numbness. The patient has a
past medical history of chronic hypertension, iatrogenic hypothyroidism, and atrial
fibrillation. A list of medications is provided by the patient’s spouse including lisinopril,
levothyroxine, and rivaroxaban. The patient’s spouse states that she was in her usual
state of health this morning but has become increasingly confused en route to the
hospital. Vitals signs are shown below. An emergent non-contrast CT of the head is
performed and shown. An immediate neurosurgical consultation is placed.

Temperature: 99.0 F (37.2 C)


Blood pressure: 170/68 mmHg
Heart rate: 58/min
Respirations: 10/min
Oxygen saturation: 99% on room air
Glasgow coma scale: 12

Which of the following best explains the cause of neurologic deficit in this patient?

⚪ A. Widespread cerebral hypoperfusion


⚪ B. Hemispheric herniation inferior to the falx cerebri
⚪ C. Diffuse shearing of white matter tracts
⚪ D. Localized shearing of cortical bridging veins
⚪ E. Acute high-impact fracture to the left pterion
≣ ⟽ ⟾
Item 1 of 1 Test Your Knowledge
◽" Mark Difficulty: ✪✪ Bootcamp.com
Question ID: 0029 Previous Next
https://commons.wikimedia.org/wiki/File:Subfalcine-herniation-001.jpg

A 67-year-old female is brought to the emergency department by paramedics following


acute onset right-sided lower extremity weakness and numbness. The patient has a
past medical history of chronic hypertension, iatrogenic hypothyroidism, and atrial
fibrillation. A list of medications is provided by the patient’s spouse including lisinopril,
levothyroxine, and rivaroxaban. The patient’s spouse states that she was in her usual
state of health this morning but has become increasingly confused en route to the
hospital. Vitals signs are shown below. An emergent non-contrast CT of the head is
performed and shown. An immediate neurosurgical consultation is placed.

Temperature: 99.0 F (37.2 C)


Blood pressure: 170/68 mmHg
Heart rate: 58/min
Respirations: 10/min
Oxygen saturation: 99% on room air
Glasgow coma scale: 12

Which of the following best explains the cause of neurologic deficit in this patient?

⚪ A. Widespread cerebral hypoperfusion


$ B. Hemispheric herniation inferior to the falx cerebri
⚪ C. Diffuse shearing of white matter tracts
⚪ D. Localized shearing of cortical bridging veins
⚪ E. Acute high-impact fracture to the left pterion

You might also like